On-line: r9о-11, гостей 6. Всего: 7 [подробнее..]
АвторСообщение



Пост N: 1
Зарегистрирован: 27.04.16
Рейтинг: 0
ссылка на сообщение  Отправлено: 27.04.16 02:54. Заголовок: Критерий выбора рабочей точки


Приветствую всех.

Решил заняться сборкой ламповых усилителей вплотную.
Ранее, лет 10-12 назад, был небольшой опыт сборки 5 усилков..., в подробности их работы, особо не вникал, но солидные люди интересовались, слушали... и продал.

Но несколько месяцев всё прочитываю литературу, и никак не могу выяснить способ выбора рабочей точки "с нуля", то есть: взять лампу, её ВАХ и... поставить точку РТ.
И меня смущает тот факт, что во многих пересмотренных схемах, РТ всегда располагается ниже тока, который заявлен в параметрах ламп (подробнее объясню в обсуждении).

Это еще не все. Будет масса интересных "моих" вычислений по этой теме и другим.
Прошу подробно объяснить данное вычисление или принцип, отнестись без скептицизма и как к новичку.
Спасибо.

Спасибо: 0 
ПрофильЦитата Ответить
Ответов - 246 , стр: 1 2 3 4 5 6 7 8 9 10 11 12 13 All [только новые]


постоянный участник




Пост N: 1086
Зарегистрирован: 28.09.09
Откуда: Украина, Черновцы
Рейтинг: 7
ссылка на сообщение  Отправлено: 27.04.16 03:14. Заголовок: Один из критериев - ..


Один из критериев - непревышение мощности рассеяния на аноде (и сетках) лампы - это к "ниже тока, который заявлен в параметрах ламп".

Главные критерии - требуемый уровень выходного сигнала (и входного тоже) и минимум искажений. Искажения считаются или в симуляторах или методами трех/пяти ординат.

Поэтому ищется точка, влево-вправо от которой (по прямой нагрузки, которая поворачивается для удовлетворения этим требованиям) выходные напряжения одинаковы при одинаковых изменениях входного напряжения (справедливо для триодов).

Или Вы хотите поделиться своим методом нахождения РТ, судя по "Будет масса интересных "моих" вычислений"? Не хочу показаться грубым, но почему-то сразу вспомнилось выражение Алекса Торреса "Прежде чем сделать открытие загляни в учебник"...

Спасибо: 0 
ПрофильЦитата Ответить





Пост N: 1064
Зарегистрирован: 20.08.13
Откуда: Россия, Москва
Рейтинг: 2
ссылка на сообщение  Отправлено: 27.04.16 03:16. Заголовок: О каких каскадах реч..


О каких каскадах речь? Предварительных, оконечных? Трансформаторных, резистивных?

Спасибо: 0 
ПрофильЦитата Ответить



Пост N: 2
Info: -
Зарегистрирован: 27.04.16
Откуда: Россия, Смоленск
Рейтинг: 0
ссылка на сообщение  Отправлено: 27.04.16 03:48. Заголовок: Речь пока о триодах ..


Речь пока о триодах в преде.

Но начну с того, что анодное сопротивление - это прямая, соединяющая две точки: питания и тока. Но возможно ли его вычислить из РТ?

И можно ли подробнее про выбор.

- Спасибо: 0 
ПрофильЦитата Ответить





Пост N: 1065
Зарегистрирован: 20.08.13
Откуда: Россия, Москва
Рейтинг: 2
ссылка на сообщение  Отправлено: 27.04.16 04:00. Заголовок: B_Aleko пишет: анод..


B_Aleko пишет:
 цитата:
анодное сопротивление... возможно ли его вычислить из РТ?

Что? Как это?

Спасибо: 0 
ПрофильЦитата Ответить



Пост N: 4
Info: -
Зарегистрирован: 27.04.16
Откуда: Россия, Смоленск
Рейтинг: 0
ссылка на сообщение  Отправлено: 27.04.16 04:43. Заголовок: Вообще то я хотел ра..


Позже нарисую и покажу на ВАХ

Вообще-то, я хотел раскрытого объяснения выбора Рабочей Точки.
Не по напряжению, не по току, не по смещению, а, сперва по визуальному взгляду на ВАХ. и при доступности относительно высокого анодного напряжения.

Зная из характеристик средние значения, можно варьировать (в разумных пределах) токами, напряжениями, смещением. но при этом соблюсти баланс разностей всего перечисленного. Как-то так.
При расчете из РТ, а не из крайних точек, (по стандартным формулам Ома), есть много интересных выводов...

Прошу обойтись без цитирования кого-либо и адекватно общаться. Ведь способы вычисления одни.

- Спасибо: 0 
ПрофильЦитата Ответить
moderator




Пост N: 3967
Зарегистрирован: 15.01.10
Откуда: Россия
Рейтинг: 18
ссылка на сообщение  Отправлено: 27.04.16 09:28. Заголовок: B_Aleko пишет: не со..


B_Aleko пишет:
 цитата:
И меня смущает тот факт, что во многих пересмотренных схемах, РТ всегда располагается ниже тока, который заявлен в параметрах ламп

Похоже, Вы сравнивали режимы с данными из даташитов на лампы, под заголовком Номинальные электрические данные.
Следует знать, что это не данные для РТ какого-либо каскада, а данные КОНТРОЛЬНОЙ точки. Именно в этой точке проверяют параметры ламп (Ri, S, µ) на заводе-изготовителе на соответствие указанным допускам. Не соответствует - брак!, поставке не подлежит. И потребитель, получающий лампы на своём предприятии, проверяет их параметры строго в этой же точке. Не укладываются в допуски - брак, вернуть изготовителю для замены, а не пожелает - предъявить ему штрафные санкции!
 цитата:
подробнее объясню в обсуждении

Излагайте, попытаемся Вас понять

ЗЫ. Такое пожелание. Если будете выкладывать свои мысли просто текстом, понять Вашу концепцию нам будет трудновато. Сопровождайте Ваши мысли рисунками, вот Вам подсказка:
http://hiend.borda.ru/?1-16-0-00000106-000-0-0-1460610681

Спасибо: 0 
ПрофильЦитата Ответить



Пост N: 5
Info: -
Зарегистрирован: 27.04.16
Откуда: Россия, Смоленск
Рейтинг: 0
ссылка на сообщение  Отправлено: 27.04.16 18:13. Заголовок: Хорошо. Чуть позже н..


Хорошо.
Чуть позже нарисую.
НО, всё-таки, хотелось бы знать о способах выбора РТ!?...



- Спасибо: 0 
ПрофильЦитата Ответить





Пост N: 1066
Зарегистрирован: 20.08.13
Откуда: Россия, Москва
Рейтинг: 2
ссылка на сообщение  Отправлено: 27.04.16 22:13. Заголовок: B_Aleko пишет: НО,..


B_Aleko пишет:
 цитата:
НО, все таки хотелось бы знать о способах выбора РТ!?...

Открываем учебник, там всё описано. Если сильно лениво, то
http://www.radiolamp.ru/shem/unch/112/

Спасибо: 0 
ПрофильЦитата Ответить



Пост N: 6
Info: -
Зарегистрирован: 27.04.16
Откуда: Россия, Смоленск
Рейтинг: 0
ссылка на сообщение  Отправлено: 28.04.16 00:06. Заголовок: Вот в том то и дело:..


Вот в том то и дело: "Отложив на оси напряжений величину напряжения источника питания (А), на оси токов величину (В):"
А как этот источник напряжения-то выбрать? может кто захочет и 100 вольт. или величину тока загнать вверх? Тогда как?
Я имею в виду критерий ВЫБОРА не относительно этих величин!
Может надо как то по другому объяснить? как?

- Спасибо: 0 
ПрофильЦитата Ответить
постоянный участник


Пост N: 883
Зарегистрирован: 25.01.10
Откуда: Минск
Рейтинг: 8
ссылка на сообщение  Отправлено: 28.04.16 01:41. Заголовок: B_Aleko Берите схем..


B_Aleko
Берите схемы удачных усей. И изучайте на ВАХ режимы, что люди выбрали. Это и есть ответ на ваши вопросы. И никого не надо будет упрашивать. Зато вопрос можно задать реальный: " Ты почему такую точку выбрал в этом месте?"

Спасибо: 0 
ПрофильЦитата Ответить



Пост N: 8
Info: -
Зарегистрирован: 27.04.16
Откуда: Россия, Смоленск
Рейтинг: 0
ссылка на сообщение  Отправлено: 28.04.16 01:47. Заголовок: я про тож и толкую! ..


И я про то же! Только никак в толк взять не могу, почему РТ внизу ВАХ?

- Спасибо: 0 
ПрофильЦитата Ответить
постоянный участник


Пост N: 884
Зарегистрирован: 25.01.10
Откуда: Минск
Рейтинг: 8
ссылка на сообщение  Отправлено: 28.04.16 01:56. Заголовок: B_Aleko 90% это комп..


B_Aleko, В 90% случаев - это компромисс с питанием. Нормальные герои лампу жарят и 500 вольт дают. Другие - используют трансформаторную нагрузку, и т.д. В каждом техническом решении свои недостатки и преимущества.

Спасибо: 0 
ПрофильЦитата Ответить



Пост N: 9
Info: -
Зарегистрирован: 27.04.16
Откуда: Россия, Смоленск
Рейтинг: 0
ссылка на сообщение  Отправлено: 28.04.16 02:51. Заголовок: Однако. Вот график. ..


Однако. Вот график. что можно по нему сказать с ваших точек зрения?
http://shot.qip.ru/00Qiah-1YLAzZMmv/



- Спасибо: 0 
ПрофильЦитата Ответить
постоянный участник


Пост N: 886
Зарегистрирован: 25.01.10
Откуда: Минск
Рейтинг: 8
ссылка на сообщение  Отправлено: 28.04.16 03:53. Заголовок: B_Aleko Ну... :sm12..


B_Aleko,
Ну...
На приложенном примере видно, что разработчик задался напряжением питания анодной цепи 200 Вольт.
Судя по наклону нагрузочной прямой, анодная нагрузка 15 кОм.
Смещение сетки лампы -1,5В.
Разработчика устроил коэффициент усиления по напряжению Ку=24.
Исходя из Ri=5.8кОм, его устроило выходное сопротивление 15*5.8/(15+5.8) = 4.2 кОм, что достаточно при средних значениях входной ёмкости выходного каскада.

ЗЫ
Мог ошибиться в расчетах.

Спасибо: 0 
ПрофильЦитата Ответить
постоянный участник




Пост N: 1090
Зарегистрирован: 28.09.09
Откуда: Украина, Черновцы
Рейтинг: 7
ссылка на сообщение  Отправлено: 28.04.16 03:58. Заголовок: Сопротивление анодно..


Сопротивление анодной нагрузки 15 кОм (200В/13 мА).
Напряжение источника питания выбрано 200В. При автоматическом смещении 198,5 В. Но никто эти десятые доли и даже 2-3 вольта анодного питания не учитывает: лампы имеют больший разброс. Поэтому считаем, что напряжение источника питания 200В.
Сопротивление анодной нагрузки 15 кОм (200В/13 мА).
Напряжение смещения около 1,5В..
Каскад линеен до выходного напряжения ±35 В, выходное сопротивление 4,8 кОм (параллельное соединение внутреннего сопротивления лампы в рабочей точке 7,1 кОм и нагрузки 15 кОм, без учета шунтирования входным сопротивлением следующего каскада.

Но возникают вопросы: а сколько надо на выходе? а что подается на вход? Я, например, применяю лампы со смещением 6-10 вольт на входе и вольт 20-30 в драйвере - голова не болит по поводу залезания сигналом на границы линейного режима.

PS. oleg_s чуток опередил, но - два подсчета дали совпадающие результаты.

Спасибо: 0 
ПрофильЦитата Ответить
администратор




Пост N: 6952
Зарегистрирован: 21.05.07
Откуда: Иркутск
Рейтинг: 24
ссылка на сообщение  Отправлено: 28.04.16 08:28. Заголовок: Немаловажное значени..


Немаловажное значение имеет выбор напряжения смещения, чтобы от уровня сигнала мгновенное значение напряжения на сетке не было ниже 0.
Иначе сеточные токи и соответственно искажения.
Если пик сигнала 1 вольт, то смещение выбирают не менее 1,5-2 вольта.
Я по крайней мере начинаю с этого.

Новый сайт http://tubeaudio.ucoz.ru/
Тел. в Иркутске: рабочий (3952) 23-52-54. моб 661-681. 8-90-25-661-681.
sergeev158(собака)mail.ru
С уважением, Сергеев Сергей Евгеньевич.
Спасибо: 0 
ПрофильЦитата Ответить



Пост N: 10
Info: -
Зарегистрирован: 27.04.16
Откуда: Россия, Смоленск
Рейтинг: 0
ссылка на сообщение  Отправлено: 28.04.16 13:09. Заголовок: Сергеев Сергей пишет..


Сергеев Сергей пишет:
 цитата:
Если пик сигнала 1 вольт, то смещение выбирают не менее 1,5-2 вольта

Вот это уже близко. Приму к сведению.

oleg_s пишет:
 цитата:
Исходя из Ri 5.8К...

Не совсем понял, как Ri получилось 5,8 кОм?
Вроде там мю=24 и S=1,6

У меня так:

мю=24,7
S=1,6
Ri=15,4
Ra=15,4
Rk=260-280 Ом
Uс (РТ)=-1,4

Положительная волна по напряжению немного меньше отрицательной.
Или не так считаю, формулы-то одни и те же.

Вопрос: возможно ли так, что внутреннее сопротивление лампы равно анодному?

- Спасибо: 0 
ПрофильЦитата Ответить
постоянный участник


Пост N: 889
Зарегистрирован: 25.01.10
Откуда: Минск
Рейтинг: 8
ссылка на сообщение  Отправлено: 28.04.16 16:27. Заголовок: B_Aleko пишет: Не с..


B_Aleko пишет:
 цитата:
Не совсем понял, как Ri получилось 5,8? Вроде там м=24 и S=1,6

http://istok2.com/data/575/
Мю = 38±6,
S = 5.9±1.8



Спасибо: 0 
ПрофильЦитата Ответить



Пост N: 13
Info: -
Зарегистрирован: 27.04.16
Откуда: Россия, Смоленск
Рейтинг: 0
ссылка на сообщение  Отправлено: 28.04.16 16:29. Заголовок: Мю 38+-6 S 5.9 +-1...


oleg_s, так это - паспортные данные, а я - по расчету в данном рисунке.

Еще вопрос.
При входном сигнале с амплитудой в 1В, по данному рисунку, сколько выходное? что-то я никак.

----------------------

В выходном каскаде хочу поставить 6П13С (их у меня аж 10 шт. все новые) или 6П31С (4 шт.).

Как можно согласовать с 6Н3П, с учетом, что положительная волна амплитуды немного меньше отрицательной?
И хотелось бы в 6П13С смещение от -4 до -8V

P.S. Есть еще вариант. имеются 6Ж32П - 12 шт. тож новые. но в пентодах пока не силен. вместо 6Н3П?

- Спасибо: 0 
ПрофильЦитата Ответить
постоянный участник




Пост N: 1092
Зарегистрирован: 28.09.09
Откуда: Украина, Черновцы
Рейтинг: 7
ссылка на сообщение  Отправлено: 28.04.16 17:33. Заголовок: Параметры вычисляютс..


B_Aleko пишет:
 цитата:
я - по расчету в данном рисунке.

1. Параметры лампы вычисляют по её ВАХ в окрестностях выбранной РТ, и желательно (т. к. параметры - величины дифференциальные) - при минимальных от нее отклонениях. По приведенному рисунку:

Ri=∆Up/∆Ip=(136-102)/(7,8-3,0)=34/4.8=7,1 кОм
S=∆Ip/∆Ug=(7,8-3,0)/(2-1)=4,8/1=4,8 мА/В
mu=∆Up/∆Ug=(136-102)/(2-1)=34/1=34

(все численные значения округлены ввиду графического исходного материала).

А коэффициент передачи (коэффициент усиления) при нагрузке 15 кОм равен:
Ку= (130-106)/(2-1)=24 (без учета шунтирования входным сопротивлением последующего каскада)

2. Для 6П13С в триодном включении, в качестве драйвера гораздо лучше применить 6Ж4, 6Ж8 (обе в триоде) или 6Г2 - вообще хорошо.

B_Aleko пишет:
 цитата:
Есть еще вариант. имеются 6Ж32П - 12 шт. тож новые. но в пентодах пока не силен. вместо 6Н3П?

Не надо 6Ж32П с током анода всего 1-2 мА нагружать на 6П13С, ранний завал ВЧ обеспечен.

Спасибо: 0 
ПрофильЦитата Ответить



Пост N: 17
Info: -
Зарегистрирован: 27.04.16
Откуда: Россия, Смоленск
Рейтинг: 0
ссылка на сообщение  Отправлено: 28.04.16 17:49. Заголовок: Ri=deltaUp/deltaIp=(..



 цитата:
Ri=∆Up/∆Ip=(136-102)/(7,8-3,0)=34/4.8=7,1 кОм

Это взято при дельтаUc с разницей в 1В?
но по формулам и из рисунка при дельтаUc 3В: (152-78)/(7,8-3,0)=74/4,8=15,4 !
в чем разница таких расчетов?

6Н3П выбрал по причине большого количества 46шт. Есть 6н1, 6н2, 6н23, 6ф1, 6ф12, 6ж1,2,3,4,5,9(63шт),10,11,32,38,44,49,51,52; 6к4, 6к13, 6с4п(2шт), 6в2п
других нет.

- Спасибо: 0 
ПрофильЦитата Ответить
moderator




Пост N: 3972
Зарегистрирован: 15.01.10
Откуда: Россия
Рейтинг: 18
ссылка на сообщение  Отправлено: 28.04.16 18:35. Заголовок: B_Aleko пишет: в чем..


B_Aleko пишет:
 цитата:
в чем разница таких расчетов?

Дело в следующем.
∆U - некоторое изменение напряжения. Можно взять и 1 В, и 3 В. Но по теории, для определения дифференциальных величин S, Ri, Мю мы должны брать в РТ бесконечно малые приращения напряжений и токов: dU и dI, тогда будет максимальная точность результатов вычислений.
Но бесконечно малые невозможно отложить на графике, и приходится использовать те кривые ВАХ, что есть на графике. Чем меньше выбранные "дельты", тем лучше, а значит, лучше выбирать соседние линии из семейства ВАХ. Опять же, если имеющийся график семейства ВАХ имеет удобный масштаб, чтобы легче было определять отрезки, спроецированные на его оси.
--------------------

to All
Для тех, кто пишет, что 6н3п - кривовата, предлагаю посмотреть ВАХи, практически снятые Клаус'ом на характериографе:



Позже Алекс Рощупкин снял ВАХи 6н3п с реальной лампы "по точкам", вот что получил:



Видим , что 6н3п - самый обычный, достаточно линейный триод.

Спасибо: 0 
ПрофильЦитата Ответить



Пост N: 19
Info: -
Зарегистрирован: 27.04.16
Откуда: Россия, Смоленск
Рейтинг: 0
ссылка на сообщение  Отправлено: 28.04.16 18:56. Заголовок: вот вроде запутался...


Вот, вроде запутался. Значит, при полном размахе величин "дельта" имеем максимальные выходные параметры? и для каждого режима усиления все справедливо?

- Спасибо: 0 
ПрофильЦитата Ответить
moderator




Пост N: 3973
Зарегистрирован: 15.01.10
Откуда: Россия
Рейтинг: 18
ссылка на сообщение  Отправлено: 28.04.16 19:01. Заголовок: "Дельтами" -..


"Дельтами" - называют маленькие кусочки линии или координат их крайних точек. При полном размахе имеем уже не "дельты", а АМПЛИТУДЫ сигнала.

Спасибо: 0 
ПрофильЦитата Ответить



Пост N: 21
Info: -
Зарегистрирован: 27.04.16
Откуда: Россия, Смоленск
Рейтинг: 0
ссылка на сообщение  Отправлено: 28.04.16 19:08. Заголовок: Пермяк пишет: "..


Пермяк пишет:
 цитата:
"Дельтами" - называют маленькие кусочки линии или координат их крайних точек. При полном размахе имеем уже не "дельты", а АМПЛИТУДЫ сигнала.

тогда где эти дельты на графике?

- Спасибо: 0 
ПрофильЦитата Ответить
постоянный участник




Пост N: 1096
Зарегистрирован: 28.09.09
Откуда: Украина, Черновцы
Рейтинг: 7
ссылка на сообщение  Отправлено: 28.04.16 21:46. Заголовок: B_Aleko пишет: тогд..


B_Aleko пишет:
 цитата:
тогда где эти дельты на графике?

Минимально возможные видимые на графике дельты возьмите из моего сообщения №1092 и найдите их на приведенном Вами графике (как, Вы этого еще не сделали?).
 цитата:
но по формулам и из рисунка при дельтаUc 3В: (152-78)/(7,8-3,0)=74/4,8=15,4!

Если брать смещение 0 и минус 3 В, то (174-62)/(15-1)=108/14= 7,7 кОм! Т. е. при расчете по амплитудам вместо дельт, как справедливо указал Пермяк, значение изменяется, но немного (зависит от лампы, конечно - у линейной лампы это изменение меньше) и не в разы...
PS. При использовании формул и рисунков не забывайте о физическом смысле рассчитываемого параметра.

Вдогонку - 6Ж51П, 6Ж52П в триоде - прекрасные лампы! 51-ая к тому же не микрофонит.

Спасибо: 0 
ПрофильЦитата Ответить



Пост N: 22
Info: -
Зарегистрирован: 27.04.16
Откуда: Россия, Смоленск
Рейтинг: 0
ссылка на сообщение  Отправлено: 28.04.16 22:00. Заголовок: ALSS пишет: Если бр..


ALSS пишет:
 цитата:
Если брать смещение 0 и минус 3 В, то (174-62)/(15-1)=108/14= 7,7 кОм!

вот окончательно запутался. напишите формулу, а с цифрами разберусь

- Спасибо: 0 
ПрофильЦитата Ответить
постоянный участник


Пост N: 893
Зарегистрирован: 25.01.10
Откуда: Минск
Рейтинг: 8
ссылка на сообщение  Отправлено: 29.04.16 01:14. Заголовок: B_Aleko Да просто в..


B_Aleko , да просто все
S = LIM ( dI(Uo,Io)/dUg(Uo,Io) ) при dUg -> 0, Ua= const

Спасибо: 0 
ПрофильЦитата Ответить



Пост N: 23
Info: -
Зарегистрирован: 27.04.16
Откуда: Россия, Смоленск
Рейтинг: 0
ссылка на сообщение  Отправлено: 29.04.16 01:56. Заголовок: oleg_s пишет: Да пр..


oleg_s пишет:
 цитата:
Да просто все

спасибо. теперь все стало на места.
То, что я считал, что Rа=200в/13мА=15,4 кОм, можно делить на 2, ввиду разницы напряжения смещения в 1В. а значит 15,4/2=7,7.
но здесь мне кажется более точнее.
Может, использовать координаты самой точки? ведь полученные данные верны для полупериода.
Тогда при положении в РТ получим еще более точное значение: берем Uo=116, Io=5,4 получаем: Ri=(Uo/Io)/2=(116/5.4)/2=10.75 Это вернее для случая при отсутствии сигнала на входе и относительно небольшого сопротивления цепи питания.

По расчетам выходит, что чем больше входной сигнал - амплитуда, тем меньше внутренне сопротивление лампы. вывод - считать при 0 амплитуды, в РТ, и при макс. значении.

- Спасибо: 0 
ПрофильЦитата Ответить
постоянный участник


Пост N: 894
Зарегистрирован: 25.01.10
Откуда: Минск
Рейтинг: 8
ссылка на сообщение  Отправлено: 29.04.16 02:31. Заголовок: B_Aleko Я дал форму..


B_Aleko, Я дал формулу крутизны S.
На графике для определения S дельта тока берется для двух соседних ветвей ВАХ по вертикали, т.е. при неизменном Ua.

Для определения Ri берется касательная к одной ветке ВАХ в выбранной точке (РТ), там дельты анодных характеристик, в пределе получается производная, графическое отображение этой производной - это котангенс угла наклона касательной к кривой в данной точке.

Спасибо: 0 
ПрофильЦитата Ответить



Пост N: 26
Info: -
Зарегистрирован: 27.04.16
Откуда: Россия, Смоленск
Рейтинг: 0
ссылка на сообщение  Отправлено: 29.04.16 03:04. Заголовок: oleg_s пишет: хор..


oleg_s, хорошо когда знаешь расчет и оперируешь научными терминами. а как быть, если только начинаешь?
а попроще...........

- Спасибо: 0 
ПрофильЦитата Ответить
постоянный участник




Пост N: 1097
Зарегистрирован: 28.09.09
Откуда: Украина, Черновцы
Рейтинг: 7
ссылка на сообщение  Отправлено: 29.04.16 03:14. Заголовок: B_Aleko, да не запут..


B_Aleko, да не запутывайте Вы себя формулами, в особенности если физический смысл их ускользает. Потрудитесь построить указанные "дельты" и "амплитуды" и вникнуть в соотношения величин.

B_Aleko пишет:
 цитата:
Uo=116, Io=5,4 получаем: Ra=(Uo/Io)/2=(116/5.4)/2=10.75

Это статическое сопротивление лампы в данной рабочей точке, да и то неправильно рассчитанное, а искомое Ri - динамическое сопротивление.
 цитата:
по расчетам выходит, что чем больше входной сигнал - амплитуда, тем меньше внутренне сопротивление лампы. вывод - считать при 0 амплитуды, в РТ, и при макс. значении.

Внутреннее сопротивление лампы от входного (и выходного тоже) сигнала не зависит - это-то Вы понимаете?
Зачем считать при нуле, если надо усиливать сигналы при отрицательном напряжении на управляющей сетке (а, забыл, под фонарем светлее...)?!

PS. А если еще и подбросить классическое "Жарьте Ваши триоды!", так вообще крышу снесёт, наверное...

Спасибо: 0 
ПрофильЦитата Ответить



Пост N: 27
Info: -
Зарегистрирован: 27.04.16
Откуда: Россия, Смоленск
Рейтинг: 0
ссылка на сообщение  Отправлено: 29.04.16 04:02. Заголовок: Спасибо всем, думаю ..


Спасибо всем, думаю что с теорией вкусную кашу не сваришь, хрустеть стеклами будет. надо на практике либо не до жарить, либо забыть до черного дыма.
на ветке останусь. может по формулам всем определюсь. в книгах одно, а на деле иначе.

- Спасибо: 0 
ПрофильЦитата Ответить
moderator




Пост N: 3974
Зарегистрирован: 15.01.10
Откуда: Россия
Рейтинг: 18
ссылка на сообщение  Отправлено: 29.04.16 09:22. Заголовок: B_Aleko пишет: с тео..


B_Aleko пишет:
 цитата:
с теорией вкусную кашу не сваришь,

Не теория виновата, а те, кто косноязычно пытается её излагать! Всегда поражался, насколько плохо "технари" излагают свои мысли на форумах...

Должен сказать, что график ВАХ, на котором Вы делаете расчёт - неудачный. Эти ВАХ, когда-то очень коряво снятые, перекочевали практически во все справочники, и при первом взгляде на них просто отталкивают от применения 6н3п в звуковой аппаратуре.
Между тем, ещё Ю.Макаров, применявший эту лампу, писал на форуме Аудиопортал, что ВАХи эти - неправильные. Я выше (пост 3972) привёл ВАХ, снятые Клаус'ом цифровым характериографом,, а также ВАХ от Алекса Рощупкина.

Спасибо: 0 
ПрофильЦитата Ответить



Пост N: 28
Info: -
Зарегистрирован: 27.04.16
Откуда: Россия, Смоленск
Рейтинг: 0
ссылка на сообщение  Отправлено: 29.04.16 13:14. Заголовок: Пермяк пишет: Не те..


Пермяк пишет:
 цитата:
Не теория виновата, а те, кто косноязычно пытается её излагать! Всегда поражался, насколько плохо "технари" излагают свои мысли на форумах...

вообще-то, я имел в виду, что на одной теории сыт не будешь, без масла с сахаром в виде практики, не вкусно будет.

чтобы быть более обстоятельным, на рисунке добавил формулы которые вычитал в книгах. и график чертил по описаниям многих книг.
от них и отталкиваюсь.
http://shot.qip.ru/00Qiah-2YLAzZMmP/

- Спасибо: 0 
ПрофильЦитата Ответить
moderator




Пост N: 3979
Зарегистрирован: 15.01.10
Откуда: Россия
Рейтинг: 18
ссылка на сообщение  Отправлено: 29.04.16 13:50. Заголовок: B_Aleko пишет: на ри..


B_Aleko пишет:
 цитата:
на рисунке добавил формулы

Каждая из этих формул имеет смысл только тогда, когда рядом с ней написано условие: при неизменной такой-то величине.
ЗЫ. Я уже начал готовить картинки, чтобы выложить здесь в теме. Ваш рисунок слишком труден.

Спасибо: 0 
ПрофильЦитата Ответить



Пост N: 30
Info: -
Зарегистрирован: 27.04.16
Откуда: Россия, Смоленск
Рейтинг: 0
ссылка на сообщение  Отправлено: 29.04.16 14:09. Заголовок: может это поможет: h..


может это поможет: http://shot.qip.ru/00Qiah-3YLAzZMmQ/
картинка прозрачная, поверх первой положить и видно будет.
этот треугольник и есть алгоритм выбора РТ. главное, что бы вершина и половина гипотенузы совпадали с линией смещения сетки.
а равносторонний треугольник можно вращать до выбора оптимальной точки. ведь как его не крути, то дельты U и I всегда будут одинаковы.
прошу не осуждать, если неправильно что то делаю, книги хорошо но люди - лучше.

вот решил на графике "Клаусмобиль" применить треугольник. особо не старался. (не осуждайте)
http://shot.qip.ru/00Qiah-2YLAzZMmR/
На графике по расчетам мю 24-25, крутизна почти 1. сопротивление в аноде 27 кОм, катодный 290 Ом, макс. мощность анодного резистора 1,4 Вт.



- Спасибо: 0 
ПрофильЦитата Ответить
постоянный участник




Пост N: 1099
Зарегистрирован: 28.09.09
Откуда: Украина, Черновцы
Рейтинг: 7
ссылка на сообщение  Отправлено: 29.04.16 16:23. Заголовок: Биссектриса - это та..


Не всегда получается выбирать нагрузку по перпендикуляру асимптоте зависимости анодного тока от анодного напряжения
при заданном напряжении смещения, да и не всегда это оптимальное решение. В линейных лампах хоть как клади нагрузку - будет все симметрично, в нелинейных - хоть тресни и ищи стабильные точки по Карпову, толку не будет (речь идет о больших сигналах).


Спасибо: 0 
ПрофильЦитата Ответить
moderator




Пост N: 3986
Зарегистрирован: 15.01.10
Откуда: Россия
Рейтинг: 18
ссылка на сообщение  Отправлено: 29.04.16 17:48. Заголовок: Проба http://f6.s.qi..


to B_Aleko
Предлагаю Вам более удобный (как мне кажется) пример для рассмотрения метода определения параметров триода.
Пока ознакомьтесь, о параметрах - чуть позже.



Можно определить коэффициент усиления Ку такого каскада. Мы видим, что если амплитуда сеточного напряжения равна 1 В, амплитуда анодного напряжения составляет
(153-95)/2=29
Тогда
Ку= Ua/Uc=29/1= 29.
(где Ua и Uc - амплитуды анодного и сеточного напряжений).

Спасибо: 0 
ПрофильЦитата Ответить



Пост N: 32
Info: -
Зарегистрирован: 27.04.16
Откуда: Россия, Смоленск
Рейтинг: 0
ссылка на сообщение  Отправлено: 29.04.16 17:50. Заголовок: Пермяк пишет: http..


Пермяк, а думаете есть смысл в уменьшении размаха сеточного напряжения?
а можно ли тогда ваш принцип расчета?
пока подумаю над рисунком.

- Спасибо: 0 
ПрофильЦитата Ответить
moderator




Пост N: 3987
Зарегистрирован: 15.01.10
Откуда: Россия
Рейтинг: 18
ссылка на сообщение  Отправлено: 29.04.16 18:00. Заголовок: B_Aleko пишет: дума..


B_Aleko, я не уменьшил размах сеточного напряжения. Для каждой полуволны - по 1 вольту запас. Вам ранее предлагали смещение -2 В, Вы согласились, я это и отразил.
А коэффициент усиления практически не зависит от того, какой величины сигнал подаётся на сетку.

Спасибо: 0 
ПрофильЦитата Ответить



Пост N: 37
Info: -
Зарегистрирован: 27.04.16
Откуда: Россия, Смоленск
Рейтинг: 0
ссылка на сообщение  Отправлено: 29.04.16 18:47. Заголовок: Пермяк пишет: Коэфф..


Пермяк пишет:
 цитата:
коэффициент усиления практически не зависит от того, какой величины сигнал подаётся на сетку.

тогда как это ВСЁ рассчитывать?
столько инфы в нете, много читаю, но в толк не возьму. и как выбрать правильные расчеты?

ЗЫ. Забыл сказать. ПРАКТИЧЕСКИЕ работы не скоро смогу воплотить. так как многое сгорело в доме (оч. давно), и приборы. а вот сейчас решил заняться из того что есть. потому и начал с теории. в которой надо разобраться, палить и фейерверки утраивать на съемной кв. не хочется.

- Спасибо: 0 
ПрофильЦитата Ответить
постоянный участник


Пост N: 899
Зарегистрирован: 25.01.10
Откуда: Минск
Рейтинг: 8
ссылка на сообщение  Отправлено: 29.04.16 19:04. Заголовок: B_Aleko пишет: тогд..


B_Aleko пишет:
 цитата:
тогда как это ВСЁ рассчитывать?

забыл сказать. ПРАКТИЧЕСКИЕ работы не скоро смогу воплотить.

Есть подозрение: вам надо микрокап освоить - будете рассчитывать в уме и на машине и сравнивать.

Спасибо: 0 
ПрофильЦитата Ответить



Пост N: 39
Info: -
Зарегистрирован: 27.04.16
Откуда: Россия, Смоленск
Рейтинг: 0
ссылка на сообщение  Отправлено: 29.04.16 19:07. Заголовок: Пермяк пишет: Ку - ..


Пермяк пишет:
 цитата:
Ку - частное от деления двух величин. Если сеточная амплитуда возрастёт в два раза, то и анодное - почти во столько же раз, и частное от деления останется таким же.

значит, в выше приведенном графике, при увеличении сеточного на 1 В, то вместо 95В будет 80, а вместо 153 - 165?

oleg_s пишет:
 цитата:
микрокап освоить

ага. знаю такую. если бы трое детей дали возможность! а так - на ходу все считаю.

- Спасибо: 0 
ПрофильЦитата Ответить
постоянный участник


Пост N: 900
Зарегистрирован: 25.01.10
Откуда: Минск
Рейтинг: 8
ссылка на сообщение  Отправлено: 29.04.16 19:37. Заголовок: B_Aleko пишет: бы т..


B_Aleko пишет:
 цитата:
если бы трое детей дали возможность!

Учите с ними! Они быстрее освоят. Зуб даю.
Моя 8ми-летка за два тыка научилась у папы в смартфоне фотки смотреть.

Спасибо: 0 
ПрофильЦитата Ответить
moderator




Пост N: 3989
Зарегистрирован: 15.01.10
Откуда: Россия
Рейтинг: 18
ссылка на сообщение  Отправлено: 30.04.16 00:48. Заголовок: B_Aleko пишет: значи..


B_Aleko пишет:
 цитата:
значит, в выше приведенном графике, при увеличении сеточного на 1 В, то вместо 95В будет 80, а вместо 153 - 165?

Да. Примерно так.

Спасибо: 0 
ПрофильЦитата Ответить





Пост N: 5
Зарегистрирован: 07.03.16
Откуда: Украина, Полтава
Рейтинг: 0
ссылка на сообщение  Отправлено: 30.04.16 02:24. Заголовок: B_Aleko пишет: тогд..


B_Aleko пишет:
 цитата:
тогда как это ВСЁ рассчитывать?


http://zzxm.narod.ru/
http://www.oldradioclub.ru/catalog/book_links_k.html
С.Н.Кризе "Усилительные устройства", 1968 год
А так же: http://radiolamp.ucoz.ru/load/literatura/lampovye_usiliteli_moshhnosti_v_rezhime_klassa_a/5-1-0-80
- скачать.


Последняя ссылка - там всё просто и понятно. Внимательно прочтите. Проще не бывает.



Спасибо: 0 
ПрофильЦитата Ответить



Пост N: 41
Info: -
Зарегистрирован: 27.04.16
Откуда: Россия, Смоленск
Рейтинг: 0
ссылка на сообщение  Отправлено: 30.04.16 02:41. Заголовок: спасибо. Кризе - об..


спасибо. Кризе - обстоятельно, но сразу с формул.
нашел в нете книгу: Гершунский Б. С. Основы электроники, 1977 г., и немного сканов из его книги 1966г.
глава про лампы, там вообще по полочкам каждый параметр.

- Спасибо: 0 
ПрофильЦитата Ответить
moderator




Пост N: 3991
Зарегистрирован: 15.01.10
Откуда: Россия
Рейтинг: 18
ссылка на сообщение  Отправлено: 30.04.16 12:34. Заголовок: http://f3.s.qip.ru/S..


B_Aleko
Мой метод определения параметров лампы в данной точке отличается от некоторых распространённых, с построениями треугольников. Считаю, что математически он более правильный и простой.






Внутреннее сопротивление (Ri) лампы вычисляем по формуле:

Ri = µ/S = 38/5,5=6,9 кОм

Спасибо: 0 
ПрофильЦитата Ответить



Пост N: 42
Info: -
Зарегистрирован: 27.04.16
Откуда: Россия, Смоленск
Рейтинг: 0
ссылка на сообщение  Отправлено: 30.04.16 14:48. Заголовок: Пермяк пишет: Мой м..


Пермяк пишет:
 цитата:
Мой метод определения параметров лампы

вот спасибо. Не мог сообразить вычисление дельты тока. брал с наклонной нагрузки анодного.

oleg_s пишет:
 цитата:
будет делать с крутизной лампы?

еще думаю. но смутные догадки, что величина влияет на полосу пропускания

- Спасибо: 0 
ПрофильЦитата Ответить



Пост N: 44
Info: -
Зарегистрирован: 27.04.16
Откуда: Россия, Смоленск
Рейтинг: 0
ссылка на сообщение  Отправлено: 30.04.16 15:24. Заголовок: если Ri=7к, Ra=27к, ..


Вопрос:
если, например, Ri=7к, Ra=27к, то тогда для чего эта разница в сопротивлениях?

- Спасибо: 0 
ПрофильЦитата Ответить



Пост N: 46
Info: -
Зарегистрирован: 27.04.16
Откуда: Россия, Смоленск
Рейтинг: 0
ссылка на сообщение  Отправлено: 30.04.16 18:24. Заголовок: форум поможет в окон..


форум поможет в окончательном вычислении результатов?
на графике есть точки. как все таки правильно высчитываются "дельты"? (график с хорошим разрешением)
http://shot.qip.ru/00Qiah-1YLAzZMn4/
можно ли для сверки результатов, рассчитать параметры по графику и приписать их к графику? и + входное и выходное напряжение.
только с питанием пока засада. в наличии пока 300В основное и, что могу для преда - это 270В.
благодарен, кто отзовется в мои трудные минуты...

- Спасибо: 0 
ПрофильЦитата Ответить
постоянный участник




Пост N: 1107
Зарегистрирован: 28.09.09
Откуда: Украина, Черновцы
Рейтинг: 7
ссылка на сообщение  Отправлено: 30.04.16 19:42. Заголовок: Пермяк Пост N: 3991 ..


Пермяк пишет:
 цитата:
Мой метод определения параметров лампы в данной точке

Это и есть классический способ определения параметров!
И без всяких к делу не относящихся треугольников и параллелепипедов.

Спасибо: 0 
ПрофильЦитата Ответить
moderator




Пост N: 3993
Зарегистрирован: 15.01.10
Откуда: Россия
Рейтинг: 18
ссылка на сообщение  Отправлено: 30.04.16 19:50. Заголовок: B_Aleko пишет: вычи..


B_Aleko пишет:
 цитата:
вычисление дельты тока. брал с наклонной нагрузки анодного.

Эти параметры никак не связаны с нагрузкой.
Кроме того, в теории сказано:

мю=dUa/dUc при неизменном токе анода Ia, то есть, вспомогательный отрезок АВ должен быть горизонтальным.

S = dIa/Uc при неизменном напряжении Ua, то есть, вспомогательный отрезок СD должен быть вертикальным.

мю=dUa/dUc при неизменном токе анода Ia, то есть, вспомогательный отрезок должен быть горизонтальным.
Наклонные линии не нужны.
 цитата:
что величина крутизны влияет на полосу пропускания

Значение S может пригодиться для выбора шунтирующего катодный резистор конденсатора (при автосмещении). То есть, действительно, S косвенно влияет и на полосу частот. Собственно, как и другие параметры - мю и Ri.
 цитата:
если Ri=7к, Ra=27к, то тогда для чего эта разница в сопротивлениях?


Для триода - чем больше Ra по отношению к Ri, тем меньше искажений и выше Ку. В идеале хорошо бы, если бы Ra было равно бесконечности

Спасибо: 0 
ПрофильЦитата Ответить



Пост N: 52
Info: -
Зарегистрирован: 27.04.16
Откуда: Россия, Смоленск
Рейтинг: 0
ссылка на сообщение  Отправлено: 01.05.16 01:58. Заголовок: вот вроде так: http:..


Вот, вроде так:



а усиление каскада?



- Спасибо: 0 
ПрофильЦитата Ответить
moderator




Пост N: 3998
Зарегистрирован: 15.01.10
Откуда: Россия
Рейтинг: 18
ссылка на сообщение  Отправлено: 01.05.16 05:57. Заголовок: B_Aleko пишет: вопр..


B_Aleko пишет:
 цитата:
вопрос: для чего надо увеличивать анодное сопротивление для триода (2-10Ra) и уменьшать для пентода (0.1-.02Ra)?

Про триод я Вам писАл чуть выше (мой пост 3993): "Для триода - чем больше Ra по отношению к Ri, тем меньше искажений и выше Ку. В идеале хорошо бы, если бы Ra было равно бесконечности"
Для пентода специально уменьшать Ra не надо, его ВАХи таковы, что Rа равное (0,1-0,2) от Ri получается среднестатистически где-то в этих пределах.

Спасибо: 0 
ПрофильЦитата Ответить
moderator




Пост N: 3999
Зарегистрирован: 15.01.10
Откуда: Россия
Рейтинг: 18
ссылка на сообщение  Отправлено: 01.05.16 08:28. Заголовок: B_Aleko пишет: а уси..


B_Aleko пишет:
 цитата:
а усиление каскада?

Ку= Ua/Ug
где:
Ua - амплитуда анодного напряжения,
Ug - амплитуда сеточного напряжения.

Попробуйте сосчитать сами, в качестве подспорья выкладываю график.
Кривые (синусоиды) напряжений при расчёте, разумеется, не рисуют, это я подрисовал Вам для лучшего понимания.





Спасибо: 0 
ПрофильЦитата Ответить



Пост N: 56
Info: -
Зарегистрирован: 27.04.16
Откуда: Россия, Смоленск
Рейтинг: 0
ссылка на сообщение  Отправлено: 01.05.16 14:21. Заголовок: Пермяк пишет: Сп..


Пермяк
Спасибо огромное за помощь и терпение. я бы еще долго разбирался.

По этому графику при входном сигнале амплитудой 1В:
Ku=(165-90)/2=37,5..

С праздниками всех!

- Спасибо: 0 
ПрофильЦитата Ответить
moderator




Пост N: 4001
Зарегистрирован: 15.01.10
Откуда: Россия
Рейтинг: 18
ссылка на сообщение  Отправлено: 01.05.16 16:21. Заголовок: С праздниками всех! ..


С праздником, коллега!

Снимать проекцию на ось напряжений надо с точек на нагрузочной прямой.

Тогда, при амплитуде на сетке Ug.m=1В, амплитуда анодного напряжения будет равна:

Ua.m=(155-100)/2=27,5 В
и усиление будет равно
Ку=27,5/1=27,5.

А при амплитуде на сетке Ug.m=2В (как на рисунке):
Ua.m=(180-70)/2=55 В
Ку=55/2=27,5

Ку с ростом входного сигнала не изменяется, о чём мы писали ранее.

Спасибо: 0 
ПрофильЦитата Ответить



Пост N: 58
Info: -
Зарегистрирован: 27.04.16
Откуда: Россия, Смоленск
Рейтинг: 0
ссылка на сообщение  Отправлено: 01.05.16 16:36. Заголовок: Пермяк пишет: Эх, з..


Пермяк пишет:
 цитата:
Снимать проекцию на ось напряжений надо с точек на нагрузочной прямой.!

только сейчас увидел.
Извиняюсь очень, не те цифры вставил, это мю.
эх, как эт я не аккуратно...

- Спасибо: 0 
ПрофильЦитата Ответить



Пост N: 60
Info: -
Зарегистрирован: 27.04.16
Откуда: Россия, Смоленск
Рейтинг: 0
ссылка на сообщение  Отправлено: 03.05.16 19:14. Заголовок: приветствую всех. у ..


приветствую всех. у меня засада с пентодами и луч. тетродами.
пентодов оч. много.
на выход хочу поставить 6п13с, пока однотакт, чтобы разобраться в расчетах.
Только почему-то встречаю его смещение в -45В. а на ВАХ только до -16В. скачал книгу "Справочник электровакуумным и полупроводниковым приборам А.М.Бройде 1962", там есть ВАХ многих ламп. но таких параметров не нашел. подскажете по ВАХ-ам?

- Спасибо: 0 
ПрофильЦитата Ответить





Пост N: 7
Зарегистрирован: 07.03.16
Откуда: Украина, Полтава
Рейтинг: 0
ссылка на сообщение  Отправлено: 04.05.16 01:50. Заголовок: B_Aleko пишет: пока..


B_Aleko пишет:
 цитата:
пока однотакт

Будете делать в триоде? Или пока - разобраться?
Ia=50...60 mA; Ua=325...330 V. Так же необходимо учесть падение напряжения питания на первичной обмотке вых. трансформатора.

B_Aleko пишет:
 цитата:
в триодном не могу найти эти ВАХ


http://radikal.ru/big/3f5650b6fc3248609e46f6d39d5f27d8]



Я пользовался этой ВАХ.

Спасибо: 0 
ПрофильЦитата Ответить



Пост N: 61
Info: -
Зарегистрирован: 27.04.16
Откуда: Россия, Смоленск
Рейтинг: 0
ссылка на сообщение  Отправлено: 04.05.16 04:04. Заголовок: омельян пишет: Буде..


омельян пишет:
 цитата:
Будете делать в триоде? Или пока - разобраться?

пока хочу разобраться в расчете по ВАХ в триодном и тетродном включении. Но в триодном не могу найти эти ВАХ.
Расчеты есть, но без наглядного графика - никак!

- Спасибо: 0 
ПрофильЦитата Ответить
moderator




Пост N: 4013
Зарегистрирован: 15.01.10
Откуда: Россия
Рейтинг: 18
ссылка на сообщение  Отправлено: 05.05.16 11:13. Заголовок: http://f2.s.qip.ru/~..


ВАХи Александра Рощупкина (слегка доработал, думаю, А.Р. не рассердится.)
Rэа - резистор, соединяющий экранную сетку с анодом.



Спасибо: 0 
ПрофильЦитата Ответить



Пост N: 62
Info: -
Зарегистрирован: 27.04.16
Откуда: Россия, Смоленск
Рейтинг: 0
ссылка на сообщение  Отправлено: 05.05.16 12:53. Заголовок: Пермяк пишет: ВАХи..


Пермяк пишет:
 цитата:
ВАХи Александра Рощупкина

Спасибо. А где взять эти ВАХи Александра Рощупкина? нашел Бройде 1962 года.

- Спасибо: 0 
ПрофильЦитата Ответить





Пост N: 1071
Зарегистрирован: 20.08.13
Откуда: Россия, Москва
Рейтинг: 2
ссылка на сообщение  Отправлено: 05.05.16 15:34. Заголовок: http://f2.s.qip.ru/w..




Спасибо: 0 
ПрофильЦитата Ответить



Пост N: 63
Info: -
Зарегистрирован: 27.04.16
Откуда: Россия, Смоленск
Рейтинг: 0
ссылка на сообщение  Отправлено: 05.05.16 18:33. Заголовок: Stan Marsh пишет: ..


Stan Marsh,
огромнейше благодарен!
А какой ток анода лампы? в параметрах написано только: в импульсе 200мА?

- Спасибо: 0 
ПрофильЦитата Ответить
moderator




Пост N: 4014
Зарегистрирован: 15.01.10
Откуда: Россия
Рейтинг: 18
ссылка на сообщение  Отправлено: 05.05.16 21:51. Заголовок: http://f5.s.qip.ru/J..


B_Aleko пишет:
 цитата:
А где взять эти ВАХи Александра Рощупкина?

У Александра был сайт на pochta.ru, где он и выкладывал снятые им характеристики. Примерно год назад почта.ру приказала долго жить, и новый владелец не торопится восстановить все сайты.



Ток, который длительно выдерживает лампа, легко определить по кривой (гиперболе) Pa+Рэ на графике для выбранного Ua.

Спасибо: 0 
ПрофильЦитата Ответить



Пост N: 65
Info: -
Зарегистрирован: 27.04.16
Откуда: Россия, Смоленск
Рейтинг: 0
ссылка на сообщение  Отправлено: 05.05.16 22:50. Заголовок: спасибо, кто откликн..


спасибо, кто откликнулся.
хотел бы изучить расчет тетродов и пентодов по графику, для драйверов и выходного.
а в однотакте проверить и триодное и пентодное включение 6П13С.

- Спасибо: 0 
ПрофильЦитата Ответить



Пост N: 471
Зарегистрирован: 24.03.12
Откуда: Россия, 25 км от Е-бурга
Рейтинг: 0
ссылка на сообщение  Отправлено: 06.05.16 18:37. Заголовок: 6п13с в триоде моя л..


6п13с в триоде - моя любимая (из дешевых) лампа.
240в, 60мА, смещение ~-25в, естевственно в триоде..
В тетроде НЕ ставлю...
Я брал Ra=5кОм, мощность меньше, и гармоники тоже. Кому нужней мощность, можно Ra=3кОм.
Наши лампы спокойно рассеивают до 18 Вт...

Спасибо: 0 
ПрофильЦитата Ответить



Пост N: 70
Info: -
Зарегистрирован: 27.04.16
Откуда: Россия, Смоленск
Рейтинг: 0
ссылка на сообщение  Отправлено: 06.05.16 21:01. Заголовок: Пермяк пишет: Для 6..


aur_100, мне кажется, что -25 - многовато. А с учетом того, что пред рассчитываю на Ку=25-28, то по графику должно совпадать вход=26В.

http://shot.qip.ru/00QjE0-4131FKD8B2/

- Спасибо: 0 
ПрофильЦитата Ответить



Пост N: 475
Зарегистрирован: 24.03.12
Откуда: Россия, 25 км от Е-бурга
Рейтинг: 0
ссылка на сообщение  Отправлено: 06.05.16 22:07. Заголовок: 6н3п не даст нужного..


6н3п не даст нужного Ку...
Я ставил 6ж4 в триоде, вых. 6п13с.

Спасибо: 0 
ПрофильЦитата Ответить
moderator




Пост N: 4026
Зарегистрирован: 15.01.10
Откуда: Россия
Рейтинг: 18
ссылка на сообщение  Отправлено: 06.05.16 22:26. Заголовок: Для каскада усиления..


B_Aleko, Вы неправильно ставите РТ.
Для каскада усиления мощности с выходным трансформатором построение ведётся не так, как для каскада с резистором в аноде.
Вот пример:



Спасибо: 0 
ПрофильЦитата Ответить



Пост N: 72
Info: -
Зарегистрирован: 27.04.16
Откуда: Россия, Смоленск
Рейтинг: 0
ссылка на сообщение  Отправлено: 06.05.16 23:29. Заголовок: Пермяк пишет: и ..


Пермяк и по этому графику выходит, что питание=512В???

- Спасибо: 0 
ПрофильЦитата Ответить





Пост N: 1073
Зарегистрирован: 20.08.13
Откуда: Россия, Москва
Рейтинг: 2
ссылка на сообщение  Отправлено: 06.05.16 23:44. Заголовок: B_Aleko пишет: по ..


B_Aleko пишет:
 цитата:
по этому графику выходит, что питание=512В???

Нет, по этому графику оно равно +300В.

Спасибо: 0 
ПрофильЦитата Ответить



Пост N: 73
Info: -
Зарегистрирован: 27.04.16
Откуда: Россия, Смоленск
Рейтинг: 0
ссылка на сообщение  Отправлено: 06.05.16 23:49. Заголовок: Stan Marsh пишет: Н..


Stan Marsh пишет:
 цитата:
Нет, по этому графику оно равно +300В.

так это справедливо для двухтакта? иначе как быть с правой полуволной сигнала?

- Спасибо: 0 
ПрофильЦитата Ответить





Пост N: 1074
Зарегистрирован: 20.08.13
Откуда: Россия, Москва
Рейтинг: 2
ссылка на сообщение  Отправлено: 07.05.16 01:30. Заголовок: B_Aleko пишет: так ..


B_Aleko пишет:
 цитата:
это справедливо для двухтакта?

Это справедливо для трансформаторного однотактного каскада.
 цитата:
как быть с правой полуволной сигнала?

Пользоваться.

...


Спасибо: 0 
ПрофильЦитата Ответить
moderator




Пост N: 4029
Зарегистрирован: 15.01.10
Откуда: Россия
Рейтинг: 18
ссылка на сообщение  Отправлено: 07.05.16 07:51. Заголовок: B_Aleko для пояснения..


B_Aleko, для пояснения, как образуется положительная полуволна в трансформаторном каскаде, привожу выдержку из статьи:
Михаил Смелянский. К вопросу о выборе минимально допустимой индуктивности..."

 цитата:
Как известно, положительная полуволна выходного сигнала в однотактном каскаде формируется засчёт энергии, накопленной в индуктивности засчёт протекающего через нее тока покоя.

Постулат: условием неискаженной передачи положительной полуволны выходного сигнала является выполнение баланса :

Qn ≥ Qi

где:
Qn - энергия, накопленная в катушке (Дж).
Qi - энергия положительной полуволны (Дж);

Таким образом, для неискаженной передачи положительной полуволны сигнала, индуктивность должна содержать достаточное количество энергии, определяемое формулой:

Qn =(Iр2 * L) / 2

где:
Ip - анодный ток покоя
L - индуктивность первичной обмотки трансформатора.



Спасибо: 0 
ПрофильЦитата Ответить



Пост N: 74
Info: -
Зарегистрирован: 27.04.16
Откуда: Россия, Смоленск
Рейтинг: 0
ссылка на сообщение  Отправлено: 09.05.16 14:44. Заголовок: Пермяк пишет: Для к..


Пермяк пишет:
 цитата:
Для каскада усиления мощности с выходным трансформатором построение ведётся не так, как для каскада с резистором в аноде.
Вот пример:

С праздником!!!
По этому примеру почти понятно.
Получается, что на первичке трансформатора почти нет падения питающего напряжения до значения в РТ. Поэтому РТ выбирается исходя от анодного. Тогда положение РТ берется по току покоя и напряжению смещения сетки.

Вопрос: зачем брать смещение, заведомо больше? такого расчета нигде не нашел.
Но это в триоде.
А как на пентодах (распространенных 6п14п, 6п15п, 6п18п, 6п43п (достались из бывшей мастерской)? Точно так же?

- Спасибо: 0 
ПрофильЦитата Ответить





Пост N: 1076
Зарегистрирован: 20.08.13
Откуда: Россия, Москва
Рейтинг: 2
ссылка на сообщение  Отправлено: 09.05.16 15:41. Заголовок: Смещение выбирается ..


Смещение выбирается так, чтобы не была превышена максимально допустимая мощность анода. Т.е., РТ должна лежать чуть ниже кривой максимально допустимой мощности, рассеиваемой на аноде.
Или, при отсутствии такой кривой на графике, должно выполняться условие:
Ua0*Ia0 ≤ Pa.max.доп.


Спасибо: 0 
ПрофильЦитата Ответить



Пост N: 75
Info: -
Зарегистрирован: 27.04.16
Откуда: Россия, Смоленск
Рейтинг: 0
ссылка на сообщение  Отправлено: 09.05.16 15:43. Заголовок: Stan Marsh пишет: С..


Stan Marsh пишет:
 цитата:
Смещение выбирается так, чтобы не превысить максимально допустимую мощность анода.

и как это по графику выбирается? там ведь пойдут области искажений!?

- Спасибо: 0 
ПрофильЦитата Ответить





Пост N: 1077
Зарегистрирован: 20.08.13
Откуда: Россия, Москва
Рейтинг: 2
ссылка на сообщение  Отправлено: 09.05.16 15:48. Заголовок: B_Aleko пишет: как ..


B_Aleko пишет:
 цитата:
как это по графику выбирается?

Посмотрел в паспорт, увидел 20Вт,
При выбранном анодном 300В, ток покоя выбираем 0,9*20/300=60мА
смотрим, что этому соответствует смещение -20В
такое и обеспечиваем.

(примерно так)


Спасибо: 0 
ПрофильЦитата Ответить



Пост N: 76
Info: -
Зарегистрирован: 27.04.16
Откуда: Россия, Смоленск
Рейтинг: 0
ссылка на сообщение  Отправлено: 09.05.16 16:40. Заголовок: Stan Marsh пишет: с..


Stan Marsh пишет:
 цитата:
смотрим, что этому соответствует смещение -20В

теперь понятно. а дальше как? по графику: http://shot.qip.ru/00Qk6h-6d3R3GHLx/

- Спасибо: 0 
ПрофильЦитата Ответить





Пост N: 1079
Зарегистрирован: 20.08.13
Откуда: Россия, Москва
Рейтинг: 2
ссылка на сообщение  Отправлено: 09.05.16 17:41. Заголовок: А дальше через эту т..


А дальше через эту точку проводят нагрузочную прямую с таким наклоном, чтобы верхний отрезок(до нуля на сетке) получился в 1,1...1,2 раза длиннее нижнего отрезка(до удвоенного смещения). По этому наклону определяем Ra(приведённое сопротивление). Это по-простому.
...

!!! Это справедливо для пентода/тетрода.


Спасибо: 0 
ПрофильЦитата Ответить
moderator




Пост N: 4035
Зарегистрирован: 15.01.10
Откуда: Россия
Рейтинг: 18
ссылка на сообщение  Отправлено: 09.05.16 18:40. Заголовок: Попробовал - этот ре..


Попробовал - этот режим никак не подойдёт.
6п13с в пентоде - явно не для звука. Для импульсных режимов. В крайнем случае - для двухтакта в режиме класса Б.

Спасибо: 0 
ПрофильЦитата Ответить



Пост N: 79
Info: -
Зарегистрирован: 27.04.16
Откуда: Россия, Смоленск
Рейтинг: 0
ссылка на сообщение  Отправлено: 10.05.16 01:05. Заголовок: Пермяк пишет: Попро..


Пермяк пишет:
 цитата:
Попробовал - этот режим никак не подойдёт.

Спасибо. но ведь ее используют, но видимо в триоде? и тогда как? продолжить график, что не вмещается?
может на этом примере можете показать: http://shot.qip.ru/00Qk6h-5d3R3GHLH/
по отзывам хороша, да и первое собирать, лучше, чем 6п14п.

- Спасибо: 0 
ПрофильЦитата Ответить
moderator




Пост N: 4038
Зарегистрирован: 15.01.10
Откуда: Россия
Рейтинг: 18
ссылка на сообщение  Отправлено: 10.05.16 09:35. Заголовок: Лампа 6п13с создана ..


Лампа 6п13с создана в начале 50-х гг. для работы в вых. каскаде строчной развёртки чёрно-белых телевизоров. Там она работала в пентодном включении. Для чего создана - для того и ВАХи сняты и приведены в справочниках. Других ВАХ производитель не дал. А вскоре были созданы и другие, более мощные лампы для такого применения.
B_Aleko пишет:
 цитата:
но ведь ее используют, но видимо в триоде?

Да, исключительно в триоде. Те, кто пишет, что она якобы в пентоде хорошо "звучит", практически никогда не выкладывают данных о её режиме и схемном включении, поэтому заявления их сомнительны.

И ещё. Применение пентодов и тетродов усложнено ещё и тем, что часто экранная сетка должна питаться от значительно меньшего напряжения, чем анодное. Обратите внимание на подпись под картинками ВАХ 6п13с и 6п15п от Бройде.

Посмотрите, как влияет напряжение экр. сетки Uc2 пентода на его ВАХи.



Есть, правда, некоторые методы "исправления" пентодных характеристик в динамическом режиме с помощью местных обратных связей. Два таких метода рассматривались на форуме в разделе Теория.
http://hiend.borda.ru/?1-13-0-00000036-000-0-0-1399180471
http://hiend.borda.ru/?1-13-0-00000038-000-0-0-1398743925

...................................

Недостающие ветви ВАХ на графиках можно дорисовывать. Очень часто приходится дорисовывать ветви промежуточные, если РТ оказывается между имеющимися на графике ветвями. При некотором навыке можно на триодном графике дорисовывать ветви справа, если уловить закономерность уменьшения расстояний между ветвями.
Линии ВАХ пентодов можно продолжать вправо, т.к. в зоне больших анодных напряжений они обычно близки к прямым линиям.

Спасибо: 0 
ПрофильЦитата Ответить



Пост N: 478
Зарегистрирован: 24.03.12
Откуда: Россия, 25 км от Е-бурга
Рейтинг: 0
ссылка на сообщение  Отправлено: 10.05.16 11:16. Заголовок: Не понимаю, зачем хо..


Не понимаю, зачем хотят пентодный(тетродный) звук от телевизионных ламп. Триодный, достаточно большая мощность, низкие искажения...

Спасибо: 0 
ПрофильЦитата Ответить



Не зарегистрирован
Зарегистрирован: 01.01.70
ссылка на сообщение  Отправлено: 10.05.16 11:44. Заголовок: Пентодный режим вооб..


Пентодный режим вообще не для звука, одни искажения, визг и хрип, если нужен итун то нужно делать выходной каскад на триоде с общей сеткой, и раскачивать понижающим трансформатором через катод.

Спасибо: 0 
Цитата Ответить



Пост N: 80
Info: -
Зарегистрирован: 27.04.16
Откуда: Россия, Смоленск
Рейтинг: 0
ссылка на сообщение  Отправлено: 10.05.16 13:58. Заголовок: Пермяк пишет: во..


Пермяк
Вот спасибо. Очень познавательно. Со временем многое пойму.
Так как за неимением графиков пентодов в триодном режиме нет, то и изучаю изначальное предназначение лампы.
Сейчас пока передо мной один вопрос, о положении РТ выходных ламп, дорисовки графика, расчет анодного и внутреннего сопротивления (в этом вся загвоздка). потом расчет транса.

aur_100 пишет:
 цитата:
Не понимаю, зачем хотят пентодный (тетродный) звук от телевизионных ламп. Триодный, достаточно большая мощность, низкие искажения...

сейчас просто изучаю теорию расчёта пентода. И что можно получить с лампы при использовании в разных режимах.

По лампе 6н3п.
Выбрал три режима с токами покоя:
7мА, сетка 2 В,
5мА, сетка 2.1В,
4мА, сетка 2В.
По расчетам сопротивлений, токов и напряжений после сборки всех трех вариантов. получил расчетные режимы.

Есть вопрос: для качества звучания какой наиболее предпочтителен режим: с меньшим током и большим анодным сопротивлением или большим током и меньшим сопротивлением?

- Спасибо: 0 
ПрофильЦитата Ответить



Пост N: 479
Зарегистрирован: 24.03.12
Откуда: Россия, 25 км от Е-бурга
Рейтинг: 0
ссылка на сообщение  Отправлено: 10.05.16 14:09. Заголовок: По мне, 7мА, а еще л..


B_Aleko пишет:
 цитата:
Выбрал три режима с токами покоя:

По мне, 7мА, а еще лучше 2,5в 9мА, по мах...

Спасибо: 0 
ПрофильЦитата Ответить



Пост N: 82
Info: -
Зарегистрирован: 27.04.16
Откуда: Россия, Смоленск
Рейтинг: 0
ссылка на сообщение  Отправлено: 10.05.16 14:24. Заголовок: aur_100 пишет: ч..


aur_100 чем это лучше?
и как это сказывается на качестве?

- Спасибо: 0 
ПрофильЦитата Ответить



Пост N: 480
Зарегистрирован: 24.03.12
Откуда: Россия, 25 км от Е-бурга
Рейтинг: 0
ссылка на сообщение  Отправлено: 10.05.16 15:42. Заголовок: КНИ меньше, качество..


КНИ меньше, качество лучше, но Ку чуть меньше...

Спасибо: 0 
ПрофильЦитата Ответить





Пост N: 1080
Зарегистрирован: 20.08.13
Откуда: Россия, Москва
Рейтинг: 2
ссылка на сообщение  Отправлено: 10.05.16 17:17. Заголовок: B_Aleko пишет: что ..


B_Aleko пишет:
 цитата:
что можно получить с лампы

Берём что-нибудь вроде 6П14П, характеристик с нагрузочными линиями в инете полно. И вникаем...вникаем.
Ещё раз:
каскад в резистором и каскад с трансформатором - это разные каскады;
триод и пентод - это разные лампы.


Спасибо: 0 
ПрофильЦитата Ответить



Пост N: 85
Info: -
Зарегистрирован: 27.04.16
Откуда: Россия, Смоленск
Рейтинг: 0
ссылка на сообщение  Отправлено: 10.05.16 19:04. Заголовок: я учился иначе. таки..


Резистивный каскад - понятен. Разобрался, смакетировал. Пока только на 6н3п, дальше другие на очереди. Все записываю на будущее.
Вот с выходными... Понятно, что транс не может быть резистором, что бы обеспечить РТ с большим падением напряжения питания на нём, потому и РТ выбирают практически анодное.

Принцип дорисовки графика до нужного напряжения из которого вычисляется анодное сопротивление - почти понятно.
Как вести расчет? Аналогично резистивному или есть нюансы?
При расчете брать полный размах при выбранном смещении?

Принцип расчета нужен и для триодного, лучевого тетрода и пентодного включения.

- Спасибо: 0 
ПрофильЦитата Ответить



Пост N: 88
Info: -
Зарегистрирован: 27.04.16
Откуда: Россия, Смоленск
Рейтинг: 0
ссылка на сообщение  Отправлено: 11.05.16 16:09. Заголовок: остается открытым во..


остается открытым вопрос:
По лампе 6н3п. Выбрал три режима с токами покоя:
7мА, сетка 2 В,
5мА, сетка 2.1В,
4мА, сетка 2В.
По расчетам сопротивлений, токов и напряжений после сборки всех трех вариантов. получил расчетные режимы.

Сам вопрос: для качества звучания какой наиболее предпочтителен режим: с меньшим током и большим анодным сопротивлением или большим током и меньшим сопротивлением?

- Не обещайте горы, всё равно подарите камушки. Спасибо: 0 
ПрофильЦитата Ответить
moderator




Пост N: 4043
Зарегистрирован: 15.01.10
Откуда: Россия
Рейтинг: 18
ссылка на сообщение  Отправлено: 11.05.16 16:30. Заголовок: Второй вариант предп..


Вариант 7 мА - предпочтительней.
Чем выше расположена РТ, тем меньше Ri, больше S. Это благоприятно влияет на качество звучания. БОльший анодный ток покоя - тоже само по себе неплохо.
Вам об этом уже писали дважды в этой теме.

Спасибо: 1 
ПрофильЦитата Ответить



Пост N: 89
Info: -
Зарегистрирован: 27.04.16
Откуда: Россия, Смоленск
Рейтинг: 0
ссылка на сообщение  Отправлено: 11.05.16 16:57. Заголовок: вот теперь ясно. гор..


вот теперь ясно. горизонтальную линию нагрузки сделать не реально, но расположить на максимально возможный угол с учетом мощности лампы, допустимых напряжений и токов.

- Не обещайте горы, всё равно подарите камушки. Спасибо: 0 
ПрофильЦитата Ответить
постоянный участник




Пост N: 1118
Зарегистрирован: 28.09.09
Откуда: Украина, Черновцы
Рейтинг: 8
ссылка на сообщение  Отправлено: 11.05.16 21:02. Заголовок: B_Aleko пишет: гори..


B_Aleko пишет:
 цитата:
горизонтальную линию нагрузки сделать не реально

Да запросто - генератор тока на транзисторах - еще 50-100 В или на лампах - еще 100-200 В добавить к питанию; посмотрите схемы Евгения Карпова из Одессы, например, его статью
http://next-tube.com/articles/drv/drv_3.pdf.

Спасибо: 0 
ПрофильЦитата Ответить



Пост N: 90
Info: -
Зарегистрирован: 27.04.16
Откуда: Россия, Смоленск
Рейтинг: 0
ссылка на сообщение  Отправлено: 11.05.16 23:25. Заголовок: ALSS пишет: Да запр..


ALSS пишет:
 цитата:
Да запросто - генератор тока на транзисторах

это пока сложно для меня.

- Не обещайте горы, всё равно подарите камушки. Спасибо: 0 
ПрофильЦитата Ответить



Пост N: 91
Info: -
Зарегистрирован: 27.04.16
Откуда: Россия, Смоленск
Рейтинг: 0
ссылка на сообщение  Отправлено: 12.05.16 00:23. Заголовок: есть небольшие сомне..


есть небольшие сомнения. как высчитать такие параметры как:

1. ...Мощность рассеяния на аноде можно определить по формуле: Pa=Ua0*Ia0 или как ранее писали здесь: 0,9*Pmax/Ea?

2. ...Желательно также проверить тепловые режимы резисторов каскада. Мощность рассеяния на анодном резисторе составляет: PRa=IaO(Uao-UaO) (тут вроде ошибка?), или же по Ому?

- Не обещайте горы, всё равно подарите камушки. Спасибо: 0 
ПрофильЦитата Ответить



Пост N: 92
Info: -
Зарегистрирован: 27.04.16
Откуда: Россия, Смоленск
Рейтинг: 0
ссылка на сообщение  Отправлено: 12.05.16 00:54. Заголовок: влияют ли на сопроти..


влияют ли на сопротивление катода Ra, Ri, Rc, если да то как? по расчету: Rk=Ug/Io. встречал дополнительные данные.

нашел такое решение расчета катодного резистора от компании "Миллард": Rk=Rk+(1/S)

- Не обещайте горы, всё равно подарите камушки. Спасибо: 0 
ПрофильЦитата Ответить



Пост N: 97
Info: -
Зарегистрирован: 27.04.16
Откуда: Россия, Смоленск
Рейтинг: 0
ссылка на сообщение  Отправлено: 12.05.16 02:13. Заголовок: нашел вот такой граф..


нашел вот такой график расчета входного сеточного сопротивления с сайта: http://tubeamplifier.narod.ru/mess073.htm (Рис. 3.10 Предельное значение сопротивления в цепи сетки (по данным компании Миллард))
только не пойму как им пользоваться

- Не обещайте горы, всё равно подарите камушки. Спасибо: 0 
ПрофильЦитата Ответить
постоянный участник




Пост N: 1119
Зарегистрирован: 28.09.09
Откуда: Украина, Черновцы
Рейтинг: 8
ссылка на сообщение  Отправлено: 12.05.16 03:10. Заголовок: B_Aleko, пока не пой..


B_Aleko пишет:
 цитата:
расчета входного сеточного сопротивления

Обратите внимание, что это расчет предельно допустимого максимального значения сопротивления утечки сетки в статическом режиме - это может пригодиться для расчета электрометрической схемы, например.
Для наших целей желательно, чтобы это сопротивление не слишком шунтировало источник сигнала или выходное сопротивление предыдущего каскада, чтобы не было их перегрузки и падения сигнала.
Вполне достаточное для практики соотношение от 4 до 10 раз для минимального значения сопротивления утечки сетки (название исторически сложившееся, придираться не следует).
Чем больше импеданс цепи, тем больше она подвержена действию наводок со стороны окружающей электромагнитной среды.
И даже не надо ПСН сюда привлекать.

Спасибо: 0 
ПрофильЦитата Ответить



Пост N: 104
Info: -
Зарегистрирован: 27.04.16
Откуда: Россия, Смоленск
Рейтинг: 0
ссылка на сообщение  Отправлено: 12.05.16 03:21. Заголовок: ALSS пишет: чтобы э..


ALSS пишет:
 цитата:
чтобы это сопротивление не слишком шунтировало источник сигнала или выходное сопротивление предыдущего каскада

но ведь должна же быть формула? а на "ощупь", как-то не хочется.

- Не обещайте горы, всё равно подарите камушки. Спасибо: 0 
ПрофильЦитата Ответить
moderator




Пост N: 4044
Зарегистрирован: 15.01.10
Откуда: Россия
Рейтинг: 18
ссылка на сообщение  Отправлено: 12.05.16 09:26. Заголовок: B_Aleko, формула ест..


B_Aleko, формула есть, но практика показала, что для разных экземпляров ламп одного и того же типа величина тока сеточной "утечки" имеет довольно значительный разброс, буквально - в разы, причём ток этот увеличивается при длительной эксплуатации лампы.
Практически же установлено, что для маломощных ламп достаточно ставить Rg=0,5÷1МОм. Чем меньше Rg, тем легче лампе, но хуже для предыдущего каскада или источника сигнала.
Для мощных выходных ламп величина Rg обязательно оговаривается в даташите, причём для применения при фиксированном смещении его величина меньше, чем при автосмещении.

B_Aleko пишет:
 цитата:
есть небольшие сомнения. как высчитать такие параметры как:

1. ...Мощность рассеяния на аноде можно определить по формуле: Pa=UaO*IaO или как ранее писали здесь: 0,9*Pmax/Ea ?

2. ...Желательно также проверить тепловые режимы резисторов каскада. Мощность рассеяния на анодном резисторе составляет: PRa=IaO(Uao-UaO) (тут вроде ошибка?), или же по Ому?

На п.1.
Первая формула - определение значения рассеиваемой мощности на аноде при выбранных Uao и Iao. Вторая формула- для определения допустимого тока покоя по известному (по паспорту) значению Pmax и напряжению на аноде в выбранной рабочей точке.

На п.2
Правильно будет так:
PRa=Iao*URa, или:
PRa=(Iao)2*Rа, или:
PRa=Iаo*(Uпит.-Ua).

Спасибо: 0 
ПрофильЦитата Ответить



Пост N: 105
Info: -
Зарегистрирован: 27.04.16
Откуда: Россия, Смоленск
Рейтинг: 0
ссылка на сообщение  Отправлено: 13.05.16 18:33. Заголовок: всем доброго дня! де..


Всем доброго дня!
Делаю первый отчет.



При сборке макета, из наличия 31 шт. ламп 6н3п, только одна оказалась в почти расчетном режиме (должно быть 125В, в итоге - 132В анодного и 2В на катоде (при питании 305В). Остальные стабильно работали при 141В анодного и 2В катода. Причём разброс режимов не превышал 0,05В в катоде и 140-142В в аноде.

Далее, для проверки качества звучания, собрал простой каскад на 6п14п. Транс с активным сопротивлением в 190 Ом (его параметры, шл-0,052(50Вт)?, при подаче 220В, на выходе две обмотки по 8В), другого пока нет. Одну подключил к колонке, вторую - в цепь катода 6п14п. Использовать пробовал ТВЗ, но звук не понравился, поставил как дроссель.

На очереди мотаю ШЛ 0,1(100Вт), с первичной в 3600 вит. с расчетами трансов пока не разобрался, но все впереди!
Замерял тремя приборами: АВО63, Ц20 и цифровым M890D. в катодах резисторы С5-5 1%, в аноде, сетках - МЛТ 5%.

Колонки не ахти, SVEN, с кевларовым диффузором 130мм, объем чуть более 12 литров. они никогда так не звучали! чисто, звонко. ставил на прослушивание много разных записей, голосовые партии прослушиваются с дыханием певцов
проверить спектры, искажения не предоставляется возможным из-за отсутствия необходимых приборов.

- Не обещайте горы, всё равно подарите камушки. Спасибо: 0 
ПрофильЦитата Ответить



Пост N: 107
Info: -
Зарегистрирован: 27.04.16
Откуда: Россия, Смоленск
Рейтинг: 0
ссылка на сообщение  Отправлено: 13.05.16 19:23. Заголовок: вот фото макета. ht..


вот фото макета.



с учетом, что собрано "на коленке" на деревянной основе, при максимальной мощности, фона и посторонних звуков абсолютно не слышно.
единственное, это бывают совсем небольшие наводки от сотового.
звучание по мощности сравнимо с моей акустикой в 35Вт, то есть приблизительно в 15-20Вт. (на слух).

- Не обещайте горы, всё равно подарите камушки. Спасибо: 0 
ПрофильЦитата Ответить



Пост N: 109
Info: -
Зарегистрирован: 27.04.16
Откуда: Россия, Смоленск
Рейтинг: 0
ссылка на сообщение  Отправлено: 14.05.16 00:20. Заголовок: несколько замечаний ..


несколько замечаний о своих ошибках:
1. анодное измерял относительно общего. а между анодом и катодом, абсолютно согласно графическому вычислению = 125-126В! (соответствие по графикам Бройде).
2. формула Rk=Ugo/Igo+(1/S)=500+0.243=(округленно) 500,25 Ом!, что достаточно не ощутимо.
3. так как с выходными каскадами еще не совсем разобрался, возможно и там есть критические ошибки. смущает смещение в 8В, даже при отсутствии дополнительной обмотки. питание 305В, между анодом и катодом 260В. перепробовал 6 ламп, результат один.

- Не обещайте горы, всё равно подарите камушки. Спасибо: 0 
ПрофильЦитата Ответить
moderator




Пост N: 4045
Зарегистрирован: 15.01.10
Откуда: Россия
Рейтинг: 18
ссылка на сообщение  Отправлено: 14.05.16 09:43. Заголовок: B_Aleko пишет: переп..


B_Aleko пишет:
 цитата:
перепробовал 6 ламп, результат один.

А это в большой степени потому, что при автосмещении резистор Rк является также элементом стабилизации тока через лампу, и при этом устраняется некоторый разброс параметров разных экземпляров ламп.
 цитата:
смущает смещение в 8В

Следует помнить, что важно не напряжение смещения, а ток через лампу. Ток через Rк (и через лампу) равен Ia+Ic2, и этот суммарный ток у Вас равен
8В/130ом=0,062А=62 мА.

Максимально допустимая мощность, рассеиваемая анодом и второй сеткой:
12+2=14 Вт
При токе 62 мА и напряжении Ua-к=260 В рассеиваемая мощность будет равна:
260*0,062=16 Вт, что превышает допустимую.
Надо подобрать Rк так, чтобы ток через него не превышал 14/260=54 мА.
Но если при 62 мА анод лампы не перегревается (посмотреть в темноте - не краснеет ли анод или 2-я сетка), то можно оставить и так. Есть любители несколько перегревать ("жарить") лампы, при этом отмечают повышение качества звука. Не знаю, относится ли это к 6п14п , не такая уж она могучая...



Спасибо: 1 
ПрофильЦитата Ответить



Пост N: 110
Info: -
Зарегистрирован: 27.04.16
Откуда: Россия, Смоленск
Рейтинг: 0
ссылка на сообщение  Отправлено: 14.05.16 13:48. Заголовок: Пермяк пишет: чтобы..


Пермяк пишет:
 цитата:
чтобы ток через него не превышал 14/260=54 мА.

да, так оно есть. поставил, проверил. Ua-k=270, Rk=110, I=0.054.
Вот только никак не пойму. транс 190 Ом. как высчитать Ra?

- Не обещайте горы, всё равно подарите камушки. Спасибо: 0 
ПрофильЦитата Ответить
moderator




Пост N: 4046
Зарегистрирован: 15.01.10
Откуда: Россия
Рейтинг: 18
ссылка на сообщение  Отправлено: 14.05.16 13:59. Заголовок: B_Aleko пишет: как в..


B_Aleko пишет:
 цитата:
как высчитать Ra?

Вам нужно высчитать, какое Ra может дать трансформатор? Тогда сообщите:
сопротивление Вашей АC, количество витков первичной обмотки и количество витков вторичной обмотки (которая подключена к динамику).

Спасибо: 0 
ПрофильЦитата Ответить



Пост N: 111
Info: -
Зарегистрирован: 27.04.16
Откуда: Россия, Смоленск
Рейтинг: 0
ссылка на сообщение  Отправлено: 14.05.16 14:06. Заголовок: увы, количество витк..


увы, количество витков в трансе неизвестно. ас 4 Ом.
сам хочу рассчитать на ОСМ 0,1, но пока не разобрался.

- Не обещайте горы, всё равно подарите камушки. Спасибо: 0 
ПрофильЦитата Ответить



Пост N: 112
Info: -
Зарегистрирован: 27.04.16
Откуда: Россия, Смоленск
Рейтинг: 0
ссылка на сообщение  Отправлено: 14.05.16 14:06. Заголовок: по лампе 6Н3П по схе..


по лампе 6Н3П по схеме

Ri=(160-90)/(0.0096-0.0014)=8540
m=70/2=35
S=8.2/2=4.1
проверяем соотношение, должно быть равно 1: (S*Ri)/m=1.0004.
Ku=(153-97)/2=28, или Ku=m*(Ra/(Ra+Ri))=29
как вычислить КНИ?

- Не обещайте горы, всё равно подарите камушки. Спасибо: 0 
ПрофильЦитата Ответить
moderator




Пост N: 4047
Зарегистрирован: 15.01.10
Откуда: Россия
Рейтинг: 18
ссылка на сообщение  Отправлено: 14.05.16 14:36. Заголовок: B_Aleko пишет: долж..


B_Aleko пишет:
 цитата:
должно быть равно 1

Ещё бы ему не быть равным единице, если одна величина найдена из двух других! Таким способом можно проверить только не допущена ли ошибка в делении-умножении.
Объективная проверка будет только в том случае, если Ri будет найдено независимо от двух других параметров.

 цитата:
как вычислить КНИ?

Это довольно сложно расписывать, особенно, если задействованы только две-три ВАХи... Дам ссылку на книгу.
Вот библиотека со свободным скачиваанием:
http://publ.lib.ru/ARCHIVES/_CLASSES/TEH_RAD/_Teh_rad.html
Находите в списке авторов Цыкин Г.С. Щёлкаете по фамилии, откроется список его работ.
Из списка выбираете Электронные усилители. 1965 г. (в красной обложке).
Открываете стр.72 и далее.

Спасибо: 0 
ПрофильЦитата Ответить



Пост N: 116
Info: -
Зарегистрирован: 27.04.16
Откуда: Россия, Смоленск
Рейтинг: 0
ссылка на сообщение  Отправлено: 14.05.16 15:13. Заголовок: Пермяк пишет: Откры..


Пермяк пишет:
 цитата:
Открываете стр.72 и далее.

скачал. спасибо

- Не обещайте горы, всё равно подарите камушки. Спасибо: 0 
ПрофильЦитата Ответить



Пост N: 117
Info: -
Зарегистрирован: 27.04.16
Откуда: Россия, Смоленск
Рейтинг: 0
ссылка на сообщение  Отправлено: 14.05.16 18:54. Заголовок: может зря спрашиваю:..


может зря спрашиваю:
еще вопрос.
если у 6п14п смещение 6В, а усиление 6н3п =28.
амплитуда 6н3п =28, а входная 6п14п 12В?

собрал выход на 6п13с, смещение -30В. звук громче, чище, но нейтральный.
входная амплитуда в 30В?
что то запутался...

- Не обещайте горы, всё равно подарите камушки. Спасибо: 0 
ПрофильЦитата Ответить
постоянный участник




Пост N: 1120
Зарегистрирован: 28.09.09
Откуда: Украина, Черновцы
Рейтинг: 8
ссылка на сообщение  Отправлено: 14.05.16 19:54. Заголовок: B_Aleko пишет: еще ..


B_Aleko пишет:
 цитата:
еще вопрос.
если у 6п14п смещение 6В, а усиление 6н3п =28.
амплитуда 6н3п =28, а входная 6п14п 12В?

При смещении 8 В максимальный размах сигнала на сетке 6П14П может быть - чтобы не заходить в сеточные токи - 8х2=16 В.
16 В делим на коэффициент усиления предыдущего каскада 28 и получаем размах входного для 6Н3П сигнала 16/28=0,57 В (т. е. 0,57/2,82=0,2 В скз).
Почему же Вы вставляете 1 В и дальше носитесь с получающимися 28 В?! Не подавайте 1 В и не будет 28 В на аноде 6Н3П.

Спасибо: 0 
ПрофильЦитата Ответить
moderator




Пост N: 4049
Зарегистрирован: 15.01.10
Откуда: Россия
Рейтинг: 18
ссылка на сообщение  Отправлено: 14.05.16 23:12. Заголовок: B_Aleko Можно попро..


B_Aleko
Можно попробовать убрать конденсатор, шунтирующий Rк лампы 6п14п. Её чувствительность уменьшится.
Также можно попробовать убрать такой же конденсатор из катода 6н3п. Ку драйвера тоже снизится, хотя и немного, думаю.
Оба варианта тщательно отслушать, поочерёдно и совместно.

B_Aleko пишет:
 цитата:
собрал выход на 6п13с, смещение -30В. звук громче, чище, но нейтральный. входная амплитуда в 30В?

Вот это - правильно. Триод в вых. каскаде - много лучше, чем пентод. Амплитуда сигнала на сетку 6п13с поступает, возможно, и меньше, чем 30 В, зато не вгоняет её в положительные значения на сетке. Отсюда и лучшее звучание. И выходной трансформатор для триода изготовить проще.

Если Вы не знаете моточных данных выходного транса, то хотя бы измерьте приблизительно его коэффициент трансформации Ктр.
Подайте на первичку напряжение U1=220В (а если есть ЛАТР, то вольт 100), и измерьте напряжение U2 на вторичной обмотке, которую будете потом подключать к динамику.
Ктр=U1/U2


Спасибо: 1 
ПрофильЦитата Ответить



Пост N: 120
Info: -
Зарегистрирован: 27.04.16
Откуда: Россия, Смоленск
Рейтинг: 0
ссылка на сообщение  Отправлено: 15.05.16 01:53. Заголовок: пока думаю, чем раск..


пока думаю, чем раскачать 6П13С. смотрю графики триодов, способных дать усиление более 28.
вместо 6П14П, поставил 6П18П со смещением в 10В. если не делать полную громкость, звучит ярче, детальней, чем 6П14П. но звук действительно пентодный... не сравнимо конечно. он жестче...

- Не обещайте горы, всё равно подарите камушки. Спасибо: 0 
ПрофильЦитата Ответить
постоянный участник




Пост N: 1121
Зарегистрирован: 28.09.09
Откуда: Украина, Черновцы
Рейтинг: 8
ссылка на сообщение  Отправлено: 15.05.16 02:46. Заголовок: Поставьте 6П43П со с..


Поставьте 6П43П со смещением 25 В. В триоде, конечно.

А зачем Вам триод с усилением более 28-ми? Ведь и так чувствительность получается 0,2 В скз, любой фонокорректор можно ко входу подключать, а сигнал от проигрывателя компакт-дисков придётся давить в 10 раз, что приводит к завалу ВЧ из-за фильтра из сопротивления регулятора уровня и входной емкости (монтажной и Миллера) каскада на 6Н3П.
6Ж52П в триоде имеет мю под 80, и 6Ж51П в триоде мало чем уступает, если речь идет о пальчиковых лампах. Октальные - и 6Н9С, и 6Г2, и 6Г7.

Кстати, прекрасная связка на 6Г2 и 6П13С (Манаков? Сергеев?) неоднократно с успехом повторена
http://rh.adsh.org.ua/search.php?search_id=1404827866


Спасибо: 0 
ПрофильЦитата Ответить





Пост N: 11
Зарегистрирован: 07.03.16
Откуда: Украина, Полтава
Рейтинг: 0
ссылка на сообщение  Отправлено: 15.05.16 03:27. Заголовок: http://rh.adsh.org.u..


http://rh.adsh.org.ua/viewtopic.php?id=2472
Усилитель собран, работает прекрасно. Режимы указаны на схеме.

Спасибо: 0 
ПрофильЦитата Ответить



Пост N: 122
Info: -
Зарегистрирован: 27.04.16
Откуда: Россия, Смоленск
Рейтинг: 0
ссылка на сообщение  Отправлено: 15.05.16 03:58. Заголовок: Пермяк пишет: измер..


ALSS пишет:
 цитата:
6Ж52П в триоде имеет мю под 80, и 6Ж51П в триоде

6ж52 есть несколько, но все б/у. 51-я - только одна.
тренировался с 6ж9п, пока не получается рассчитывать по пентодным графикам. должно быть усиление более 45, а в итоге работает тише, чем триод.
А режим 6н3п менять не хочется (приберегу куда-нибудь), уж сильно точно встали параметры и по графическим расчетам КНИ =0,02%
вот 6г2 и 6н9 - собрал бы, но нет таких.

Пермяк пишет:
 цитата:
измерьте приблизительно его коэффициент трансформации Ктр

спасибо. так и сделаю.

- Не обещайте горы, всё равно подарите камушки. Спасибо: 0 
ПрофильЦитата Ответить



Пост N: 123
Info: -
Зарегистрирован: 27.04.16
Откуда: Россия, Смоленск
Рейтинг: 0
ссылка на сообщение  Отправлено: 15.05.16 04:06. Заголовок: ALSS пишет: Поставь..


ALSS пишет:
 цитата:
Поставьте 6П43П со смещением 25 В. В триоде

Подумаю. График ВАХ нашёл. Но в толк не возьму: для 14-й питание 300, ток 45-50мА, см -6-8В.
А на графике 43-й, при -25В - максимально 235В питание. Это уменьшать питание?

- Не обещайте горы, всё равно подарите камушки. Спасибо: 0 
ПрофильЦитата Ответить
moderator




Пост N: 4050
Зарегистрирован: 15.01.10
Откуда: Россия
Рейтинг: 18
ссылка на сообщение  Отправлено: 15.05.16 10:58. Заголовок: B_Aleko пишет: Это ..


B_Aleko пишет:
 цитата:
Это уменьшать питание?

Конечно. Если Вы подбираете к драйверу выходной каскад (да ещё - по напряжению его смещения!), т.е. решаете задачу наоборот, то и источник питания придётся менять, да и выходной транс, скорее всего -тоже.



Спасибо: 1 
ПрофильЦитата Ответить



Пост N: 481
Зарегистрирован: 24.03.12
Откуда: Россия, 25 км от Е-бурга
Рейтинг: 0
ссылка на сообщение  Отправлено: 15.05.16 11:02. Заголовок: Поставьте драйвером ..


Поставьте драйвером к 6п13с лампу 6н7с, лучше "Фотон", забудете другие...

Спасибо: 0 
ПрофильЦитата Ответить



Пост N: 124
Info: -
Зарегистрирован: 27.04.16
Откуда: Россия, Смоленск
Рейтинг: 0
ссылка на сообщение  Отправлено: 15.05.16 13:32. Заголовок: Пермяк пишет: Подай..


Пермяк пишет:
 цитата:
Если Вы не знаете моточных данных выходного транса

нашел данные транса в нете:
Трансформатор Т2.2025.06
(220В n=1500 d=0.2mm,)
((16.7+/-0.8В)x2, n=230.d=0.41mm )
(22-23=6.5+/-0.3В, n=45 d=0.33mm)

- Не обещайте горы, всё равно подарите камушки. Спасибо: 0 
ПрофильЦитата Ответить





Пост N: 12
Зарегистрирован: 07.03.16
Откуда: Украина, Полтава
Рейтинг: 0
ссылка на сообщение  Отправлено: 15.05.16 14:32. Заголовок: http://rh.adsh.org.u..


http://rh.adsh.org.ua/viewtopic.php?id=2472&p=2
Очень жаль, что у Вас нет 6Г2. Звучание порадовало бы Вас. Я пробовал 6Ж4 и 6Ж8, но звучание было хуже. Пентоды не понравились. Даже в триоде.
Вы сравните ВАХ 6Г2 и 6Н3П. Какая из них более линейная.

Спасибо: 0 
ПрофильЦитата Ответить



Пост N: 125
Info: -
Зарегистрирован: 27.04.16
Откуда: Россия, Смоленск
Рейтинг: 0
ссылка на сообщение  Отправлено: 15.05.16 14:59. Заголовок: омельян пишет: Очен..


омельян пишет:
 цитата:
Очень жаль, что у Вас нет 6Г2.

интересно.
Она на "Истоке" есть, надо бы заказать.
Дети идею корпуса усилителя и колонок подкинули. даже не ожидал. а под них нужны лампы такие на вид "старинные". вот теперь на выход надо что то фигурное.
только сначала надо разобраться с расчетом трансформатора и подкрепить знания в расчете выходных ламп.

- Не обещайте горы, всё равно подарите камушки. Спасибо: 0 
ПрофильЦитата Ответить



Пост N: 482
Зарегистрирован: 24.03.12
Откуда: Россия, 25 км от Е-бурга
Рейтинг: 0
ссылка на сообщение  Отправлено: 15.05.16 15:10. Заголовок: Если ставить с высок..


Если ставить с высоким мю, то 6г7, усиление поменьше, но зато Ri меньше, усиления с 6п13с хватает, можно пробовать отключать Ск.

Спасибо: 0 
ПрофильЦитата Ответить





Пост N: 13
Зарегистрирован: 07.03.16
Откуда: Украина, Полтава
Рейтинг: 0
ссылка на сообщение  Отправлено: 15.05.16 15:32. Заголовок: B_Aleko пишет: над..


B_Aleko пишет:
 цитата:
надо разобраться с расчетом трансформатора...

Если хотите, я могу дать Вам расчёт вых. трансформатора на ОСМ-0,16. Получается очень хорошо. Вых.мощность 4 Вт. Этого хватает "с головой" для домашнего прослушивания.

Спасибо: 0 
ПрофильЦитата Ответить



Пост N: 126
Info: -
Зарегистрирован: 27.04.16
Откуда: Россия, Смоленск
Рейтинг: 0
ссылка на сообщение  Отправлено: 15.05.16 15:57. Заголовок: омельян пишет: Если..


омельян пишет:
 цитата:
Если хотите, я могу дать Вам расчёт вых. трансформатора

вот это интересно. на этой основе можно ведь разобраться как рассчитывать?

- Не обещайте горы, всё равно подарите камушки. Спасибо: 0 
ПрофильЦитата Ответить





Пост N: 14
Зарегистрирован: 07.03.16
Откуда: Украина, Полтава
Рейтинг: 0
ссылка на сообщение  Отправлено: 15.05.16 16:38. Заголовок: Основу я Вам уже дав..


Основу я Вам уже давал - книга Кризе. Дайте И-мейл. Отправлю Вам книги.
См. ЛС.


Спасибо: 0 
ПрофильЦитата Ответить



Пост N: 128
Info: -
Зарегистрирован: 27.04.16
Откуда: Россия, Смоленск
Рейтинг: 0
ссылка на сообщение  Отправлено: 15.05.16 18:37. Заголовок: misteraleko@gmail.co..


misteraleko@gmail.com

- Не обещайте горы, всё равно подарите камушки. Спасибо: 0 
ПрофильЦитата Ответить
moderator




Пост N: 4051
Зарегистрирован: 15.01.10
Откуда: Россия
Рейтинг: 18
ссылка на сообщение  Отправлено: 15.05.16 19:28. Заголовок: B_Aleko В той книге ..


B_Aleko, В той книге Г.С. Цыкина, что Вы уже скачали, есть раздел по расчёту выходных трансформаторов.
Кризе у хайэндщиков давно не котируется.

B_Aleko пишет:
 цитата:
нашел данные транса в нете:
Трансформатор Т2.2025.06
(220В n=1500 d=0.2mm,)
(16.7+/-0.8В)x2, n=230.d=0.41mm )
(22-23=6.5+/-0.3В, n=45 d=0.33mm)

Это что, Вы в своём макете применяете в качестве выходника сетевой понижающий транс 220в ? Без немагнитного зазора?

Спасибо: 0 
ПрофильЦитата Ответить





Пост N: 16
Зарегистрирован: 07.03.16
Откуда: Украина, Полтава
Рейтинг: 0
ссылка на сообщение  Отправлено: 15.05.16 19:43. Заголовок: Да, согласен. Но в д..


Пермяк пишет:
 цитата:
Кризе у хайэндщиков давно не котируется.

Да, согласен. Но в данном случае - это очень даже подойдёт. Практикой подтверждено.

Спасибо: 0 
ПрофильЦитата Ответить



Пост N: 129
Info: -
Зарегистрирован: 27.04.16
Откуда: Россия, Смоленск
Рейтинг: 0
ссылка на сообщение  Отправлено: 15.05.16 20:13. Заголовок: Пермяк пишет: Это ч..


Пермяк пишет:
 цитата:
Это что, Вы в своём макете применяете в качестве выходника сетевой понижающий транс 220в ? Без немагнитного зазора?

к сожалению..., но ведь работает уже три дня, не греется, лампы в норме.

- Не обещайте горы, всё равно подарите камушки. Спасибо: 0 
ПрофильЦитата Ответить
moderator




Пост N: 4052
Зарегистрирован: 15.01.10
Откуда: Россия
Рейтинг: 18
ссылка на сообщение  Отправлено: 15.05.16 20:42. Заголовок: А с чего бы ему грет..


А с чего бы ему греться-то? Поверьте, "не греется" - не самый положительный параметр выходника.
В, общем, раз коллега омельян за Вас взялся - удачи, и Кризе - Вам в помощь!

Спасибо: 1 
ПрофильЦитата Ответить





Пост N: 17
Зарегистрирован: 07.03.16
Откуда: Украина, Полтава
Рейтинг: 0
ссылка на сообщение  Отправлено: 15.05.16 20:44. Заголовок: Пожалуйста, намотайт..


Пожалуйста, намотайте вых. трансформатор по правильным расчётам и выберите правильные режимы. Ведь иначе звук будет непредсказуемым, что подрывает основы высококачественного звучания ламповой аппаратуры.
Не помню, кто сказал: "Нужно делать хорошо, плохо у нас и так получается".

to Пермяк: Я просто подумал, что на данном этапе Кризе вполне достаточно. Цыкин - корифей.

Спасибо: 0 
ПрофильЦитата Ответить



Пост N: 130
Info: -
Зарегистрирован: 27.04.16
Откуда: Россия, Смоленск
Рейтинг: 0
ссылка на сообщение  Отправлено: 15.05.16 21:24. Заголовок: омельян пишет: ..


омельян пишет:
 цитата:
"Нужно делать хорошо, плохо у нас и так получается"

надо делать плохо, у нас и так хорошо получается. - антоним.

Вообще-то, благодаря данному форуму, многое изучил, есть книги, формулы...
на 6п13С, откладываю в долгий ящик. не нравится.
А вообще-то, 6н3п очень хороша. звучит прекрасно при разных режимах, главное - подобрать точку РТ по минимуму искажений.
В запасе еще есть
3 шт. 6п6с (б/у),
6 шт. 6п3с (б/у),
5 шт. - 1540 (вроде аналог 6п3с, б/у),
6п31С- 10 шт. (б/у) - только вряд ли с них будет толк. Зато хорошо потренируюсь в графиках и вычислениях.
6с33с - 4шт. (прямо с завода парные. давно очень заказывал). на них вряд ли скоро что то сделаю...

Надо не только для себя стараться, а стараться для семьи. Привнося удовольствие стремлением свой окружающий мир сделать ярким и незабываемым!

- Не обещайте горы, всё равно подарите камушки. Спасибо: 0 
ПрофильЦитата Ответить





Пост N: 18
Зарегистрирован: 07.03.16
Откуда: Украина, Полтава
Рейтинг: 0
ссылка на сообщение  Отправлено: 15.05.16 21:34. Заголовок: B_Aleko пишет: на 6..


B_Aleko пишет:
 цитата:
на 6п13С, откладываю в долгий ящик. не нравится.

А вот это зря. Лампа очень линейная. В триоде. Научитесь смотреть на ВАХ ламп.

Спасибо: 0 
ПрофильЦитата Ответить



Пост N: 483
Зарегистрирован: 24.03.12
Откуда: Россия, 25 км от Е-бурга
Рейтинг: 0
ссылка на сообщение  Отправлено: 15.05.16 22:47. Заголовок: С сетевым трансформа..


С сетевым трансформатором вместо выходного делайте сами из чего хотите и как хотите...

Спасибо: 0 
ПрофильЦитата Ответить



Пост N: 232
Зарегистрирован: 09.04.13
Откуда: Россия, Россошь Воронежской обл.
Рейтинг: 0
ссылка на сообщение  Отправлено: 15.05.16 22:51. Заголовок: омельян для прослу..


омельян для прослушки симф. музыки может 4 вт. и хватит , но без барабанов , они там редкость .А вот чтото приличное послушать .... можно , но без объемных низов.Прослушайте последний альбом Гилмора ! Пока не навалил ватт 20 на щиты4а32 баса неастоящего не услышал ! Попробуйте !

Спасибо: 0 
ПрофильЦитата Ответить
постоянный участник




Пост N: 1122
Зарегистрирован: 28.09.09
Откуда: Украина, Черновцы
Рейтинг: 8
ссылка на сообщение  Отправлено: 16.05.16 00:56. Заголовок: юрий 1958, у омельян..


юрий 1958, у омельян есть его работы двухтактные моноблоки на 6С33С, он знаком с повышенной мощностью...

омельян пишет:
 цитата:

B_Aleko пишет:
цитата:
на 6п13С, откладываю в долгий ящик. не нравится.

А вот это зря. Лампа очень линейная. В триоде. Научитесь смотреть на ВАХ ламп.

Полностью поддерживаю. Все части данного сообщения от омельян.

Ну, B_Aleko, из "маломощных" остается 6П41С в триоде, если крутиться около 4 Вт.

Но использование сетевого трансформатора без зазора указывает на занятие процессом без представления о конечном результате, как это ни печально. В электрическом смысле. В музыкальном, т. е. для прослушивания музыки - знакомые музыканты слышат все что их интересует на любом звуковоспроизводящем агрегате (и в любом состоянии, но это уже другой вопрос).
Если Вы обладаете такими способностями - я Вам завидую, мне же нужен хороший звуковоспроизводитель.

Спасибо: 0 
ПрофильЦитата Ответить



Пост N: 131
Info: -
Зарегистрирован: 27.04.16
Откуда: Россия, Смоленск
Рейтинг: 0
ссылка на сообщение  Отправлено: 16.05.16 02:05. Заголовок: я знаю, что много де..


я знаю, что много делаю ошибок. но без практики они - пшик. пусть пройдет время, в котором обрету истину, и возьму опыт людей.

- Не обещайте горы, всё равно подарите камушки. Спасибо: 0 
ПрофильЦитата Ответить
moderator




Пост N: 4053
Зарегистрирован: 15.01.10
Откуда: Россия
Рейтинг: 18
ссылка на сообщение  Отправлено: 16.05.16 09:25. Заголовок: О выходном трансформ..


О выходном трансформаторе.
Очень хотелось бы прикинуть, а какую нагрузку Вы дали лампе посредством этого выходника, что получили от неё такой чудесный звук?
Вы пишете:
 цитата:
нашел данные транса в нете:
Трансформатор Т2.2025.06
(220В n=1500 d=0.2mm,)
((16.7+/-0.8В)x2, n=230.d=0.41mm )
(22-23=6.5+/-0.3В, n=45 d=0.33mm)

, но при этом не сообщаете, которую из вторичек подключили к динамику, а которую - под катод?
И какие размеры имеет сердечник этого трансформатора?



Спасибо: 0 
ПрофильЦитата Ответить



Пост N: 135
Info: -
Зарегистрирован: 27.04.16
Откуда: Россия, Смоленск
Рейтинг: 0
ссылка на сообщение  Отправлено: 16.05.16 13:45. Заголовок: для сравнения исполь..


Пермяк, извините, но я не писал, что звук чудесный, просто получил хорошее звучание. Для сравнения использовал исполнителей "Пентатоникс", у них приличные голосовые партии.
https://music.yandex.ru/artist/1225394/similar
а также середины прошлого века: Dee Clark

По трансформатору:
обмотки
n=1500 d=0.2mm, - первичка
n=230 d=0.41mm - на динамик
n=45 d=0.33mm - катодная обмотка
размеры сердечника
у1=20, у2=25, b=10, h=40.

слушаю на нем сейчас Bobby Bland, не супер конечно, но достойное звучание блюза.
https://music.yandex.ru/artist/3099

- Не обещайте горы, всё равно подарите камушки. Спасибо: 0 
ПрофильЦитата Ответить



Пост N: 137
Info: -
Зарегистрирован: 27.04.16
Откуда: Россия, Смоленск
Рейтинг: 0
ссылка на сообщение  Отправлено: 16.05.16 18:47. Заголовок: вопрос по ВАХам. к п..


вопрос по ВАХам.
к примеру есть 6п14п, у нее на графике пентодный режим при напряжении 2 сетки = 250В. режим ее работы установил в точности.
а вот 6п18п нигде нет при таком напряжении сетки. как ее перерисовать (чтобы высчитать режим в пентодном включении. оч. хочу разобраться в пентодных режимах.)

- Не обещайте горы, всё равно подарите камушки. Спасибо: 0 
ПрофильЦитата Ответить





Пост N: 1082
Зарегистрирован: 20.08.13
Откуда: Россия, Москва
Рейтинг: 2
ссылка на сообщение  Отправлено: 16.05.16 20:46. Заголовок: B_Aleko пишет: 6п18..


B_Aleko пишет:
 цитата:
6п18п нигде нет при таком напряжении сетки.

А она и не предназначена для "такого" напряжения.
170В - это её предел. Не считайте разработчиков ламп глупее себя, они дают оптимальные режимы.
...
А для триодного включения картинок полно.

Спасибо: 0 
ПрофильЦитата Ответить



Пост N: 138
Info: -
Зарегистрирован: 27.04.16
Откуда: Россия, Смоленск
Рейтинг: 0
ссылка на сообщение  Отправлено: 16.05.16 21:08. Заголовок: Stan Marsh пишет: ..


Stan Marsh пишет:
 цитата:
...они дают оптимальные режимы.

я и не считаю. но многие лампы не предназначены для звука и в разных режимах. ведь разработчики тоже под это их не разрабатывали...

- Не обещайте горы, всё равно подарите камушки. Спасибо: 0 
ПрофильЦитата Ответить





Пост N: 1083
Зарегистрирован: 20.08.13
Откуда: Россия, Москва
Рейтинг: 2
ссылка на сообщение  Отправлено: 16.05.16 21:35. Заголовок: B_Aleko пишет: мног..


B_Aleko пишет:
 цитата:
многие лампы не предназначены для звука и в разных режимах.

Про 99% ламп уже давно всё известно. Остальные неинтересны или просто недоставаемы.
...


Спасибо: 0 
ПрофильЦитата Ответить
постоянный участник




Пост N: 1123
Зарегистрирован: 28.09.09
Откуда: Украина, Черновцы
Рейтинг: 8
ссылка на сообщение  Отправлено: 16.05.16 21:52. Заголовок: B_Aleko пишет: По т..


B_Aleko пишет:
 цитата:
По трансформатору:
обмотки
n=1500 d=0.2mm, - первичка
n=230 d=0.41mm - на динамик
n=45 d=0.33mm - катодная обмотка
размеры сердечника
У1=20, у2=25, b=10, h=40.

4,5 кв. см сердечник (kст=0,9) - ватта на 2 хватит, ну, 3.

Ктр=230/(1500+45)=0,15^2=0,022
4 Ом/0,022=180 Ом приведенного к аноду сопротивления нагрузки?! Издеваетесь над лампой (и не только над нею).
Может, 230 витков в катодной обмотке (около 13% - допустимо), а 45 - на динамик?
Тогда Ктр=45/(1500+230)=0,026^2=0,00068 и приведенная анодная нагрузка 4/0,00068=5,9 кОм - о, это уже похоже на необходимое лампе.

Спасибо: 0 
ПрофильЦитата Ответить





Пост N: 19
Зарегистрирован: 07.03.16
Откуда: Украина, Полтава
Рейтинг: 0
ссылка на сообщение  Отправлено: 17.05.16 03:51. Заголовок: Да, вроде как нашли ..


ALSS пишет:
 цитата:
4,5 кв. см сердечник (kст=0,9) - ватта на 2 хватит, ну, 3.

Да, вроде как нашли консенсус. Нужен сердечник от ОСМ-0,16. Будем искать.

Спасибо: 0 
ПрофильЦитата Ответить
moderator




Пост N: 4058
Зарегистрирован: 15.01.10
Откуда: Россия
Рейтинг: 18
ссылка на сообщение  Отправлено: 17.05.16 08:48. Заголовок: B_Aleko пишет: мног..


B_Aleko пишет:
 цитата:
многие лампы не предназначены для звука и в разных режимах. ведь разработчики тоже под это их не разрабатывали...

6П18П - особая лампа. Она разработана так, что при относительно малых напряжениях (Uaо=Uс2о=150v) она способна давать относительно большой ток анода.
В той же библиотеке http://publ.lib.ru/ARCHIVES/_CLASSES/TEH_RAD/_Teh_rad.html
скачайте: Терещук Р.М. "Справочник...", 1962, и со стр. 402 и далее прочитайте §6 о схемах, в которых она могла применяться.

ЗЫ. Нескромный вопрос: а осциллограф у Вас есть?
Понятно, что конечным результатом является ЗВУК, но без осциллографа оценить работу усилителя (даже по расчётам), на мой взгляд - невозможно.

Спасибо: 0 
ПрофильЦитата Ответить



Пост N: 139
Info: -
Зарегистрирован: 27.04.16
Откуда: Россия, Смоленск
Рейтинг: 0
ссылка на сообщение  Отправлено: 18.05.16 18:02. Заголовок: Добрый день. Позже о..


Добрый день.
Позже отпишусь по вопросам и вычислениям.
Верстаю очередной номер журнала. Много работы. думаю на неделю.

А по 6п18п, при питании 170-190 В ток покоя получается выше, а значит и подмагничивание транса будет больше.

- Не обещайте горы, всё равно подарите камушки. Спасибо: 0 
ПрофильЦитата Ответить



Пост N: 140
Info: -
Зарегистрирован: 27.04.16
Откуда: Россия, Смоленск
Рейтинг: 0
ссылка на сообщение  Отправлено: 20.05.16 18:47. Заголовок: Stan Marsh пишет: А..


Stan Marsh пишет:
 цитата:
А она и не предназначена для "такого" напряжения.

выкраивал время, чтобы экспериментировать и отрисовывать график лампы (черно-серые линии соответствует графику Бройде).
при увеличении напряжении второй сетки до 290 В, график строился таким образом, первый результат:



при второй сетке в 190 В, второй результат:



для расчета принимал различные амплитуды входного сигнала.
при имеющемся трансформаторе с омическим сопротивлением первички в 99 Ом, получился результат, близкий второму варианту.
вот график результата (синим указано использование транса):



и последнее, есть транс ТВЗ 1-6, его сопротивление 274 Ом, и результат оказался значительно плачевный, и при 190 В, и при 290 В. звук - ужас.

- Не обещайте горы, всё равно подарите камушки. Спасибо: 0 
ПрофильЦитата Ответить



Пост N: 142
Info: -
Зарегистрирован: 27.04.16
Откуда: Россия, Смоленск
Рейтинг: 0
ссылка на сообщение  Отправлено: 20.05.16 19:17. Заголовок: вот схема. немного и..


вот схема. немного изменена и с учетом выходного транса. собрана по синему графику ВАХ



уменьшил усиление первой лампы.
повторил схему на другом силовом, и выходном таком же трансе не подбирая лампы и детали, "на бум", результат оказался очень и очень хорош.

- Не обещайте горы, всё равно подарите камушки. Спасибо: 0 
ПрофильЦитата Ответить





Пост N: 20
Зарегистрирован: 07.03.16
Откуда: Украина, Полтава
Рейтинг: 0
ссылка на сообщение  Отправлено: 21.05.16 00:05. Заголовок: При таком раскладе у..


При таком раскладе у Вас вторая сетка работает как анод.
Напряжение на 2 сетке не должно превышать напряжение на аноде.

Спасибо: 0 
ПрофильЦитата Ответить



Пост N: 144
Info: -
Зарегистрирован: 27.04.16
Откуда: Россия, Смоленск
Рейтинг: 0
ссылка на сообщение  Отправлено: 21.05.16 01:05. Заголовок: омельян пишет: При ..


омельян пишет:
 цитата:
При таком раскладе у Вас вторая сетка работает как Анод

Отчего же она так работает? Параметры не превышены.
 цитата:
Напряжение на 2 сетке не должно превышать напряжение на аноде.

В литературе не встречал таких ограничений. Есть много примеров подключения второй сетки напрямую к питанию, в том числе - и в справочнике для 6п18п:



Или что-то пропускаю? Два полных дня работает, можно на пару сек прикоснуться, жаром не пышет (намного меньше греется, чем 6п14п).
миллиамперметр в катоде показывает стабильно 58-59 мА.
у соседа усилитель pioneer a 50, так сравнили (при приблизительно одинаковой громкости). сосед будет продавать свой...(шутка, оч понравился звук, может закажет ламповичёк. но он любит большую громкость).

- Не обещайте горы, всё равно подарите камушки. Спасибо: 0 
ПрофильЦитата Ответить





Пост N: 1085
Зарегистрирован: 20.08.13
Откуда: Россия, Москва
Рейтинг: 2
ссылка на сообщение  Отправлено: 21.05.16 02:09. Заголовок: B_Aleko пишет: я не..


омельян пишет:
 цитата:
Напряжение на 2 сетке не должно превышать напряжение на аноде.

При типовой схеме включения выходного пентода напряжение второй сетки всегда больше напряжения анода на величину падения напряжения на активном сопротивлении первичной обмотки выходного трансформатора. В пределах 10В. Это нормально.

Спасибо: 0 
ПрофильЦитата Ответить



Пост N: 147
Info: -
Зарегистрирован: 27.04.16
Откуда: Россия, Смоленск
Рейтинг: 0
ссылка на сообщение  Отправлено: 21.05.16 03:04. Заголовок: омельян пишет: По и..


По искажениям.



на графике видно. нижний хвост красной линии совсем немного выступает за линию -13. так эту линейность (что бы был линейный участок), с помощью изменения тока покоя,
можно сместить и вместо 60 мА, выставить 58 мА.
тогда смещение будет не -7,00, а -7,05 - -7,1 В к примеру. и не подавать на вход лампы размах более 13 В. у меня чуть более 13 В.

По блоку питания:



Транс ТС-180, перемотанный кем-то, брал с рынка.
При выключенных всех приборах (в общем - перед сном), шума и фона НЕТ!

- Не обещайте горы, всё равно подарите камушки. Спасибо: 0 
ПрофильЦитата Ответить





Пост N: 22
Зарегистрирован: 07.03.16
Откуда: Украина, Полтава
Рейтинг: 0
ссылка на сообщение  Отправлено: 21.05.16 03:46. Заголовок: Ну, вот и хорошо. Те..


Ну, вот и хорошо. Теперь выберите правильную ВАХ (рабочую точку) для драйверной лампы.

Спасибо: 0 
ПрофильЦитата Ответить



Пост N: 150
Info: -
Зарегистрирован: 27.04.16
Откуда: Россия, Смоленск
Рейтинг: 0
ссылка на сообщение  Отправлено: 21.05.16 17:50. Заголовок: омельян пишет: Ну, ..


омельян пишет:
 цитата:
Ну, вот и хорошо. Теперь выберите правильную ВАХ (рабочую точку) для драйверной лампы.

вот действительно, поспешишь... извиняюсь, подкорректировал график 6Н3П.



у лампы 6Н3П хорошую линейность можно получить при смещении -2, и... при -4, практически абсолютная.
Заметил одну деталь: драйвер лучше запитывать напряжением выше, чем выходной каскад. так как больше динамический диапазон и большая линейность.

- Не обещайте горы, всё равно подарите камушки. Спасибо: 0 
ПрофильЦитата Ответить
moderator




Пост N: 4059
Зарегистрирован: 15.01.10
Откуда: Россия
Рейтинг: 18
ссылка на сообщение  Отправлено: 22.05.16 00:28. Заголовок: B_Aleko пишет: есть..


B_Aleko пишет:
 цитата:
есть транс ТВЗ 1-6, его сопротивление 274 Ом, и результат оказался значительно плачевный, и при 190 В, и при 290 В. звук - ужас.

ТВЗ1-6 - выходник для ДВУХТАКТНЫХ схем вых. каскада, а значит - в сердечнике отсутствует немагнитный зазор, и при применении в однотакте его сердечник будет работать в области насыщения, то есть - никак.
Приблизительный размер зазора lз определяют по формуле:



Толщина немагнитной прокладки равна lз/2.
Требуется сердечник ТВЗ1-6 перебрать "встык" и проложить немагнитную прокладку.
На практике толщину прокладки уточняют подбором по максимальной индуктивности первичной обмотки.

Вот данные ТВЗ1-6 и значения Ra, которые он может дать при разных нагрузках:

http://shot.qip.ru/00Buq9-6IRG4yxUQ/

Спасибо: 0 
ПрофильЦитата Ответить



Пост N: 157
Info: -
Зарегистрирован: 27.04.16
Откуда: Россия, Смоленск
Рейтинг: 0
ссылка на сообщение  Отправлено: 26.05.16 03:58. Заголовок: проверил все свои 6н..


Всем вечер добрый.
Проверил в макете у лампы 6н3п ранее оговоренную РТ на ВАХ: смещение -4В, питание 275В, ток 3мА. результат реально ошеломил. Такая динамика, детальность. И почему эту лампу считают не подходящей? Она способна на большее, если внимательно к ней отнестись.

Проверил все свои 6н3п в количестве 28шт. Есть новые и б/у. Результат оказался на 90% "в точку". При других режимах разброс составляет намного больше, совпадение уже менее 60%. (по графикам Бройде).

- Не обещайте горы, всё равно подарите камушки. Спасибо: 0 
ПрофильЦитата Ответить
moderator




Пост N: 4068
Зарегистрирован: 15.01.10
Откуда: Россия
Рейтинг: 18
ссылка на сообщение  Отправлено: 26.05.16 08:16. Заголовок: B_Aleko пишет: ране..


B_Aleko пишет:
 цитата:
ранее оговоренную РТ на ВАХ: смещение -4В, питание 275В, ток 3мА

Коллега, мой Вам совет: когда выкладываете данные РТ, то указывать надо напряжение анод-катод или просто напряжение на аноде, а не "питание".
Обозначения лучше применять следующие:
Ea - напряжение питания анодной цепи каскада;
Ua0 (или Uao) - напряжение анод-катод в режиме покоя (в точке РТ);
Ua - напряжение анод-катод в любой рассматриваемой точке.
Тогда все будут понимать, о чём Вы пишете.

--------------------------------
График из Бройде, который Вам всё время подсовывают коллеги :) , для оценки 6н3п плохо пригоден. Я давал Вам приличные, вполне информативные ВАХи от Клауса и Алекса, даю ещё одну:







А по этому графику видно, например, что при смещении -4 вольта, Ri в 2 с лишним раза больше, и S в три раза меньше, чем при -2В.

Спасибо: 0 
ПрофильЦитата Ответить



Пост N: 491
Зарегистрирован: 24.03.12
Откуда: Россия, 25 км от Е-бурга
Рейтинг: 0
ссылка на сообщение  Отправлено: 26.05.16 12:33. Заголовок: B_Aleko, лампа 6н3п ..


B_Aleko, лампа 6н3п - хуже некуда, ее ставят в УВЧ, гетеродины и пр., но никак не качественные УНЧ...

Спасибо: 0 
ПрофильЦитата Ответить
администратор




Пост N: 6974
Зарегистрирован: 21.05.07
Откуда: Иркутск
Рейтинг: 24
ссылка на сообщение  Отправлено: 26.05.16 13:32. Заголовок: aur_100 пишет: ламп..


aur_100 пишет:
 цитата:
лампа 6н3п - хуже не куда, ее ставят в УВЧ, гетеродины и пр., но никак не качественные УНЧ...

Не согласен. И не только я, но многие, в том числе и Ю.Макаров.

Новый сайт http://tubeaudio.ucoz.ru/
Тел. в Иркутске: рабочий (3952) 23-52-54. моб 661-681. 8-90-25-661-681.
sergeev158(собака)mail.ru
С уважением, Сергеев Сергей Евгеньевич.
Спасибо: 0 
ПрофильЦитата Ответить



Пост N: 493
Зарегистрирован: 24.03.12
Откуда: Россия, 25 км от Е-бурга
Рейтинг: 0
ссылка на сообщение  Отправлено: 26.05.16 14:41. Заголовок: Сколько я не пытался..


Сколько я ни пытался, все равно - "транзисторное" звучание, даже в корректоре с малыми сигналами, имхо...

Спасибо: 0 
ПрофильЦитата Ответить



Пост N: 158
Info: -
Зарегистрирован: 27.04.16
Откуда: Россия, Смоленск
Рейтинг: 0
ссылка на сообщение  Отправлено: 26.05.16 14:46. Заголовок: aur_100 пишет: Скол..


aur_100 пишет:
 цитата:
Сколько я ни пытался, все равно - "транзисторное" звучание, даже в корректоре с малыми сигналами, имхо...

здесь не согласен.
Отличия ощутимы при сравнении с хорошим каменным. электролит в катоде не ставлю. максимум до 2 мкф пленочный.

- Не обещайте горы, всё равно подарите камушки. Спасибо: 0 
ПрофильЦитата Ответить





Пост N: 1087
Зарегистрирован: 20.08.13
Откуда: Россия, Москва
Рейтинг: 2
ссылка на сообщение  Отправлено: 26.05.16 14:59. Заголовок: B_Aleko пишет: 2 мк..


B_Aleko пишет:
 цитата:
2 мкф

Равносильно его отсутствию.

Спасибо: 0 
ПрофильЦитата Ответить



Пост N: 159
Info: -
Зарегистрирован: 27.04.16
Откуда: Россия, Смоленск
Рейтинг: 0
ссылка на сообщение  Отправлено: 26.05.16 15:06. Заголовок: Stan Marsh пишет: Р..


Stan Marsh пишет:
 цитата:
Равносильно его отсутствию.

большинство в иностранных и старых схем вообще конд отсутствует.

- Не обещайте горы, всё равно подарите камушки. Спасибо: 0 
ПрофильЦитата Ответить





Пост N: 1088
Зарегистрирован: 20.08.13
Откуда: Россия, Москва
Рейтинг: 2
ссылка на сообщение  Отправлено: 26.05.16 15:17. Заголовок: Отсутствие конденсат..


Отсутствие конденсатора=ООС,
малая ёмкость=частотно зависимая ООС,
большая=отсутствие ООС.

Спасибо: 0 
ПрофильЦитата Ответить



Пост N: 160
Info: -
Зарегистрирован: 27.04.16
Откуда: Россия, Смоленск
Рейтинг: 0
ссылка на сообщение  Отправлено: 28.05.16 05:03. Заголовок: журнал сдал в печать..


журнал сдал в печать. если кому интересно можете посмотреть: http://ru.calameo.com/read/003954547325294038531
я, дизайнер-верстальщик. если кому надо брошюры, буклеты и т.д., обращайтесь. мой сайт:

http://misteraleko.wix.com/misteraleko

По теме: пока не переберу режимы 6н3п, к другой лампе не перейду. И пусть я потрачу много времени на каждую, но знать буду, какая лампа может подойти в каждом конкретном случае, и как правильно выбрать её режим.

- Не обещайте горы, всё равно подарите камушки. Спасибо: 0 
ПрофильЦитата Ответить
постоянный участник




Пост N: 1134
Зарегистрирован: 28.09.09
Откуда: Украина, Черновцы
Рейтинг: 8
ссылка на сообщение  Отправлено: 28.05.16 19:29. Заголовок: B_Aleko пишет: Пока..


B_Aleko пишет:
 цитата:
Пока не переберу режимы 6н3п, к другой не перейду.

Зачем тратить время на мельчайшие детали режимов?! Поверьте, Вы потом пожалеете о затраченном на ненужные подробности времени. Задачу надо решать крупными мазками, ну а потом шлифовать.
Позволю привести собственный пример - если бы я влезал в тонкости движения электронов и абсолютно все влияющие факторы, я бы не разработал несколько сотен устройств от контактных групп до автоматизированных измерительных систем и в серийное производство пошли бы не десятки, а едва единицы изделий.
Я понимаю, что Вы более чем на 20 лет младше по возрасту, но о времени - если оно ценно - задумываться надо было еще 20 лет назад.
Простите за стариковское брюзжание, но слушайте музыку - если она Вам интересна - на сейчас работающей системе и улучшайте ее звучание, если чувствуете необходимость.
Или повторите - не думая! как ни странно это звучит - удачную разработку и идите дальше, уже имея успешный опыт за плечами.

Спасибо: 0 
ПрофильЦитата Ответить



Пост N: 161
Info: -
Зарегистрирован: 27.04.16
Откуда: Россия, Смоленск
Рейтинг: 0
ссылка на сообщение  Отправлено: 03.06.16 02:15. Заголовок: ALSS пишет: ...иди..


ALSS пишет:
 цитата:
...идите дальше, уже имея успешный опыт за плечами

время ценно, но без времени не будет опыта. спасибо.



- Не обещайте горы, всё равно подарите камушки. Спасибо: 0 
ПрофильЦитата Ответить



Пост N: 162
Info: -
Зарегистрирован: 27.04.16
Откуда: Россия, Смоленск
Рейтинг: 0
ссылка на сообщение  Отправлено: 03.06.16 02:39. Заголовок: в цепи катода имеетс..


В цепи катода имеется некое сопротивление, которое создает местную ОС. его шунтируют емкостью.

Вопрос: выбор емкости определяется частотой среза по НЧ? или чтобы полностью исключить ОС?

При использовании формулы Xc=1/(2п*f*C), выбрал нижнюю частоту 35-40 Гц, емкость составила 4,4 - 4,7 мкф, и реактивное сопротивление емкости 982 Ом.
Так как сейчас использую лампу с катодным сопротивлением в 1 кОм, возможно ли, что они компенсируют как то друг друга, имея практически одинаковое сопротивление? Резистор по постоянному напряжению, а конденсатор по переменному. какой будет коэффициент местной ОС?

- Не обещайте горы, всё равно подарите камушки. Спасибо: 0 
ПрофильЦитата Ответить
постоянный участник




Пост N: 1139
Зарегистрирован: 28.09.09
Откуда: Украина, Черновцы
Рейтинг: 8
ссылка на сообщение  Отправлено: 03.06.16 03:08. Заголовок: 1. Все классические ..


1. Все классические расчеты приведены по уровню напряжения минус 3 дБ (0,707), что в конечном итоге дает уменьшение мощности в два раза. Этого достаточно для озвучивания, но не для воспроизведения музыки, для воспроизведения музыки желательно иметь падение мощности не более 0,5-1 дБ на заданной - нижней или верхней - частоте.

2. Одним из критериев, ранее (до второй мировой) никак не рассматривавшихся в быту, является фазовый сдвиг на НЧ. Приемлемым считается значение не более 10 градусов на 40 Гц, что приводит к нижней на уровне минус 3 дБ 40/(2pi)=6,3 Гц - вот на этой частоте реактивное сопротивление конденсатора катодной цепи не должно превышать 10% от сопротивления катодного резистора, что для 1 кОм дает 100 Ом и 250 мкФ (перепроверьте, пожалуйста, считал на ходу к койке).

3. Гораздо более жесткое требование выдвигает Макаров Ю. А. - это спад импульса длительностью 1 с не более 0,17 дБ и отсюда С=50хI/U, что для 1 кОм и 10 мА требует 50*0,01/10=0,05 Ф (50000 мкФ). В реальной практике достаточно одной десятой этой величины.

4. Все вышеприведенное является не более чем моим вкусовым предпочтением. И мне ни разу не было плохо. Мало того, позволяло избавиться от "сверчково-цикадного" звучания юго-восточных усилителей любителям европейской классической музыки и всемирного рока.

5. А считать коэффициенты обратной связи? Ее не должно быть. Ну разве что местная в выходном каскаде.

Спасибо: 1 
ПрофильЦитата Ответить



Пост N: 163
Info: -
Зарегистрирован: 27.04.16
Откуда: Россия, Смоленск
Рейтинг: 0
ссылка на сообщение  Отправлено: 03.06.16 04:14. Заголовок: ALSS пишет: ...Пр..


ALSS пишет:

 цитата:
...Приемлемым считается значение не более 10 градусов на 40 Гц...

Как это высчитывается? Для меня пока это темный лес. ограничение до 10% что бы завал был на 3дб? а если больше ограничить нижнюю частоту. извиняюсь за пока некомпетентность в данном вопросе.

Почти в точку. пересчитал. можно 220+47 мкф при 99.35 Ом.
6,3 Гц, 265 мкф, 100.097 Ом.

- Не обещайте горы, всё равно подарите камушки. Спасибо: 0 
ПрофильЦитата Ответить
moderator




Пост N: 4093
Зарегистрирован: 15.01.10
Откуда: Россия
Рейтинг: 18
ссылка на сообщение  Отправлено: 03.06.16 10:51. Заголовок: B_Aleko пишет: Как э..


B_Aleko пишет:
 цитата:
Как это высчитывается?

Но Вы ведь уже рассчитали минимально необходимую величину Ск.
Поясню.
По формуле, выложенной Александром, Вы определили Ск при условии, что на частоте 6,3 Гц синусоидальный сигнал уменьшится до 0,707 от уровня на средних частотах, т.е. уменьшится в 1/0,707=1,41 раз. Это в децибелах составит спад уровня на 3 дБ.
При такой величине Ск, если подать на вход каскада сигнал типа меандр частотой 40 Гц, то спад вершин прямоугольных импульсов меандра не превысит 10%. Ну, есть такая найденная эмпирически норма, определённая "на слух", с которой, кстати, далеко не все хайэндщики согласны.

Оговорюсь, что формула эта несколько приблизительна, но вполне достаточна для практических целей.
Классический расчёт величины Ск сложен, особенно для пентода. Я мог бы предложить Вам ознакомиться с ним в имеющейся у Вас "красной" книге Г.С.Цыкина (стр. 304-307 и далее), но не буду этого делать :).

Теперь - о главном.
В усилителе, как правило, несколько каскадов, минимум 2. И если каждый каскад будет давать спад 3дБ, то на выходе усилителя спады эти просуммируются, и дадут недопустимый для нас общий спад уровня на НЧ.
По этой причине (да ещё из-за трудностей с точными книжными расчётами), на практике, величину Cк для предватительных каскадов выбирают много больше расчётной, в 10-20 и более раз. Да и для вых. каскада - тоже. И долго спорят, какой фирмы-изготовителя следует покупать эти конденсаторы :). Частенько нелюбовь к конденсаторам народ преодолевает разными неклассическими решениями: ставят в катод батарейку, либо стабилитрон, либо светодиод, и т.д. Но это - совсем другая история...

Спасибо: 0 
ПрофильЦитата Ответить



Пост N: 164
Info: -
Зарегистрирован: 27.04.16
Откуда: Россия, Смоленск
Рейтинг: 0
ссылка на сообщение  Отправлено: 03.06.16 14:53. Заголовок: Пермяк пишет: на ча..


Пермяк пишет:
 цитата:
на частоте 6,3 Гц синусоидальный сигнал уменьшится на 3 дБ.

хорошо, но если мне нужен спад ниже 40 Гц? для уменьшения габаритов выходного транса.

- Не обещайте горы, всё равно подарите камушки. Спасибо: 0 
ПрофильЦитата Ответить
постоянный участник




Пост N: 1141
Зарегистрирован: 28.09.09
Откуда: Украина, Черновцы
Рейтинг: 8
ссылка на сообщение  Отправлено: 03.06.16 16:37. Заголовок: В случае сужения пол..


В случае сужения полосы сигнала снизу для уменьшения габаритов выходного трансформатора просто подставляется другая частота в расчеты и обязательно вводится ограничение полосы перед/между каскадами, чтобы на этот уменьшенный трансформатор не пришли частоты ниже его рабочего диапазона и не ввели его сердечник в насыщение. Таким образом пытаются делать усилители для мидбаса или для ШП с поддержкой снизу. Ну и ПРО-системы озвучивания.
Вопросы задержек/опережения сигнала в полосах трогать не будем.
Конечно же следует учитывать эффект накопления ограничений, о котором пишет Пермяк, вследствие увеличения порядка фильтра. Для озвучивания этот эффект полезен.
Однако тут вступает в действие субъективный эффект - чем сильнее ограничена полоса усилителя снизу, тем мельче масштаб воспроизводимой музыки.
Для музыки лучше ограничивать - по напряжению! Не по полосе - входной сигнал и работать на меньшей мощности, но в максимально широкой полосе.
О том, что это мое субъективное мнение, напоминать далее не стОит?

Пермяк
пишет:
 цитата:
меандр частотой 40 Гц, то спад вершин прямоугольных импульсов меандра не превысит 10%

Нет, к сожалению, для 10% спада вершины на частоте меандра 40 Гц нижняя частота д. б. гораздо ниже, 0,16*0,1/0,0125=1,28 Гц.
Впрочем, в данном случае это мелочи второго порядка малости.

Спасибо: 0 
ПрофильЦитата Ответить
moderator




Пост N: 4094
Зарегистрирован: 15.01.10
Откуда: Россия
Рейтинг: 18
ссылка на сообщение  Отправлено: 03.06.16 20:44. Заголовок: Да, прошу прощения, ..


Да, прошу прощения, ошибся:

при Fн(-3дБ)=6,3 Гц скол импульса меандра частотой 40 Гц равен 50%.

Спасибо: 0 
ПрофильЦитата Ответить
moderator




Пост N: 4096
Зарегистрирован: 15.01.10
Откуда: Россия
Рейтинг: 18
ссылка на сообщение  Отправлено: 04.06.16 11:24. Заголовок: B_Aleko пишет: мне ..


B_Aleko пишет:
 цитата:
мне нужен спад ниже 40 Гц... для уменьшения габаритов выходного транса.

Коллега, если бы Вы сразу задали вопрос именно так, то сразу получили бы конкретный ответ:
для этой цели Ск как правило не применяется.

Предположим, что мы постепенно уменьшаем частоту синус-сигнала от СЧ до НЧ, и следим за уровнем на выходе каскада.
Можно подобрать ёмкость Ск так, что на какой-то частоте, скажем 30 Гц, спад будет равен 3дБ. Но если мы и дальше будем уменьшать частоту, то с некоторого момента увидим, что спад постепенно уменьшается, и на какой-то частоте спада вообще не будет. Просто потому, что на этой частоте ёмкостное сопротивление конденсатора Ск уже очень большое, он практически не шунтирует Rк, и усиление сигнала каскадом (хотя и охваченным ООС) сохраняется вплоть до 0 Гц.

Примеры вида АЧХ на НЧ при наличии Ск.



Начало спада и его прекращение зависят от исходных параметров лампы и соотношения величин Rа и Rк, и зависимость эта достаточно сложная.

Вывод: ожидать от Ск эффективной работы в качестве фильтра не следует.

Спасибо: 0 
ПрофильЦитата Ответить



Пост N: 165
Info: -
Зарегистрирован: 27.04.16
Откуда: Россия, Смоленск
Рейтинг: 0
ссылка на сообщение  Отправлено: 04.06.16 13:30. Заголовок: Пермяк пишет: получ..


Пермяк пишет:
 цитата:
получили бы конкретный ответ:

как-то в начале не подумал о трансе.
тогда: при спаде усиления, к примеру в 3 дб., в драйвере, можно настроить последующую цепочку из Rc и Cp тоже со спадом аналогичной частоты. и получим уже в 6 дб.?

- Не обещайте горы, всё равно подарите камушки. Спасибо: 0 
ПрофильЦитата Ответить



Пост N: 166
Info: -
Зарегистрирован: 27.04.16
Откуда: Россия, Смоленск
Рейтинг: 0
ссылка на сообщение  Отправлено: 04.06.16 13:45. Заголовок: при Rc=220 кОм и Cp=..


при Rc=220 кОм и Cp=22нФ, спад при частоте 32,88 Гц.
если правильно, тогда на сколько дб?

- Не обещайте горы, всё равно подарите камушки. Спасибо: 0 
ПрофильЦитата Ответить
moderator




Пост N: 4097
Зарегистрирован: 15.01.10
Откуда: Россия
Рейтинг: 18
ссылка на сообщение  Отправлено: 04.06.16 13:47. Заголовок: Совершенно верно. Но..


На пост 165:
На этой частоте суммарный спад, обусловленный этими двумя цепочками, будет 6 дБ.
Но на более низких частотах крутизна спада уменьшится.

Спасибо: 0 
ПрофильЦитата Ответить



Пост N: 167
Info: -
Зарегистрирован: 27.04.16
Откуда: Россия, Смоленск
Рейтинг: 0
ссылка на сообщение  Отправлено: 04.06.16 13:51. Заголовок: то есть, уровень на ..


то есть, уровень на частоте снизится и останется без изменений вплоть до 0Гц?

- Не обещайте горы, всё равно подарите камушки. Спасибо: 0 
ПрофильЦитата Ответить
moderator




Пост N: 4098
Зарегистрирован: 15.01.10
Откуда: Россия
Рейтинг: 18
ссылка на сообщение  Отправлено: 04.06.16 13:58. Заголовок: Нет. Цепочка CрRc по..


Нет. Цепочка CрRc по-прежнему будет давать спад, и при 0 Гц (постоянный ток!) не пропустит ничего :)

B_Aleko пишет:
 цитата:
при Rc=220 кОм и Cp=22нФ, спад при частоте 32,88 Гц. если правильно, тогда на сколько дб?

Если имеете в виду только эту цепочку, то о ней всё сказано здесь:
http://hiend.borda.ru/?1-13-0-00000093-000-0-0-1453553729

Спасибо: 0 
ПрофильЦитата Ответить



Пост N: 168
Info: -
Зарегистрирован: 27.04.16
Откуда: Россия, Смоленск
Рейтинг: 0
ссылка на сообщение  Отправлено: 13.06.16 18:19. Заголовок: Добрый день. У меня ..


Добрый день.
У меня возникло сомнение о величине:
Внутреннее сопротивление Ri - сопротивление лампы переменному току. Определяется как отношение изменения анодного напряжения к изменению анодного тока при неизменных напряжениях на остальных электродах. Измеряется в кОм. Внутреннее сопротивление может быть определено по значениям крутизны характеристики S и коэффициента усиления M: Ri=M/S

Тогда чем отличается сопротивление постоянному току от сопротивления по переменному току?


- Не обещайте горы, всё равно подарите камушки. Спасибо: 0 
ПрофильЦитата Ответить
moderator




Пост N: 4103
Зарегистрирован: 15.01.10
Откуда: Россия
Рейтинг: 18
ссылка на сообщение  Отправлено: 13.06.16 19:04. Заголовок: B_Aleko Здравствуйте..


B_Aleko Здравствуйте!
Вы задали очень интересный вопрос.
Допускаю, что с математикой у Вас не очень, поэтому попробую на этот теоретический вопрос ответить доступным (как мне кажется) языком.

Во-первых, неверны сами по себе выложенные Вами определения.
Вот как надо:
1. Сопротивление лампы постоянному току - это сопротивление в заданной (или выбранной) точке на семействе статических! ВАХ лампы. Определяется по закону Ома для пост. тока для неизменных Ua и Ia .Этому сопротивлению даже буквы специальной не присвоено, настолько оно не нужно при расчётах.

2. Ri - это сопротивление, проявлямое лампой в динамическом режиме, т.е. - при подключенной к ней нагрузке и изменении Ua и Ia
при движении рабочей точки по нагрузочной линии (в нашем случае - по прямой Ra).

Образно можно описать процесс так.
Под воздействием изменения Uc рабочая точка пересекает линии ВАХ, "наталкиваясь на них поочерёдно. При этом ей кажется, что она движется по плоскости, сплошь заполненной бесконечным множеством линий ВАХ, которые имеют примерно одинаковый наклон. И переменный ток через лампу "видит" сопротивление, которое равно dU/dI.

Что такое это самое dU/dI, Вы уже знаете: оно и есть Ri.

Если моё пояснение что-то просветило...
А если нет - то придётся углубляться в математику.
Посмотрим.

Спасибо: 0 
ПрофильЦитата Ответить
moderator




Пост N: 4109
Зарегистрирован: 15.01.10
Откуда: Россия
Рейтинг: 18
ссылка на сообщение  Отправлено: 14.06.16 16:09. Заголовок: to B_Aleko Я удалил ..


to B_Aleko
Я удалил из темы весь ранее следовавший за предыдущим постом флейм.

Вы проводите какие-то исследования, касающиеся параметров лампы? Хорошо.
Когда закончите эту работу, милости просим сюда, выложить результаты.
Желательно на понятном русском языке, чтобы не пришлось Вас переспрашивать.

Спасибо: 0 
ПрофильЦитата Ответить



Пост N: 180
Info: -
Зарегистрирован: 27.04.16
Откуда: Россия, Смоленск
Рейтинг: 0
ссылка на сообщение  Отправлено: 14.06.16 16:21. Заголовок: хорошо. согласен. То..


хорошо. согласен.
Только моя просьба остается в силе. конфиденциально, в личку контакты.

- Не обещайте горы, всё равно подарите камушки. Спасибо: 0 
ПрофильЦитата Ответить



Пост N: 182
Info: -
Зарегистрирован: 27.04.16
Откуда: Россия, Смоленск
Рейтинг: 0
ссылка на сообщение  Отправлено: 15.06.16 04:32. Заголовок: Хотелось бы вернутьс..


Хотелось бы вернуться к вопросу о величине Ri. Это же тема о теории.

Пермяк пишет:
 цитата:
1. Сопротивление лампы постоянному току - это сопротивление в заданной (или выбранной) точке на семействе статических! ВАХ лампы.

Это то же сопротивление лампы? но как быть с физикой? ведь не существует разных сопротивлений для двух величин (под переменный и постоянный ток). тем более в одном баллоне.

- Не обещайте горы, всё равно подарите камушки. Спасибо: 0 
ПрофильЦитата Ответить



Пост N: 266
Зарегистрирован: 06.04.13
Рейтинг: 0
ссылка на сообщение  Отправлено: 15.06.16 04:45. Заголовок: B_Aleko пишет: Сопр..


B_Aleko,
Вам уже ответил Пермяк:
 цитата:
Этому сопротивлению даже буквы специальной не присвоено, настолько оно не нужно при расчётах.

B_Aleko пишет:
 цитата:
но как быть с физикой?

Учить.
 цитата:
ведь не существует разных сопротивлений для двух величин (под переменный и постоянный ток).

Ещё как существует.

Учиться никогда не поздно, и никому не вредно. Спасибо: 0 
ПрофильЦитата Ответить
moderator




Пост N: 4110
Зарегистрирован: 15.01.10
Откуда: Россия
Рейтинг: 18
ссылка на сообщение  Отправлено: 15.06.16 11:26. Заголовок: Попытаюсь ещё раз. ..


Попытаюсь ещё раз.

Подключим к выводам накала лампы полагающееся ей по паспорту напряжение.
Подключим между сеткой и катодом лампы источник напряжения 3В "минусом" к сетке.
Между катодом и анодом подключим регулируемый источник напряжения "минусом" к катоду, и установим на его выходе напряжение 150 вольт.

Согласно графику семейства ВАХ ток через ламу будет равен 5 мА.
Если взять отношение абсолютных значений напряжения и тока, получим сопротивление пост. току:
R=U/I=150/5=30 кОм.
Проведём прямую АВ. Отметим, что для любой точки на этой прямой вычисленное R всегда будет равно 30 кОм. Т.е., получили сопротивление лампы постоянному току. Куда применить при вычислениях - неизвестно.



Увеличим напряжение анодного источника до 165 вольт. Т.е., дадим приращение
∆Ua=165-150=15 В
Под воздействием приращения напряжения произойдёт приращение анодного тока с 5 до 7 мА:
∆Ia=7-5=2 мА
Если возьмём отношение этих относительных приращений, получим величину:
Ri=∆Ua/∆Ia =15/2= 7,5 В/мА.
Поскольку В/мА=кОм, то есть Ri выражается в единицах сопротивления, то величине Ri дали название ВНУТРЕННЕЕ СОПРОТИВЛЕНИЕ лампы. В нашем случае, для точки А:
Ri=7,5 кОм.
Таким образом имеем два похожих названия для разных величин, сильно отличающихся одна от другой как по сути, так и в числовом выражении.

ЗЫ. Кстати, описанным под рисунком методом и измеряют Ri.

ЗЫ.ЗЫ. Поскольку ВАХа лампы не является прямой линией, то, РАЗУМЕЕТСЯ, что в разных её точках величина Ri разная: чем ниже на линии ВАХ находится точка, тем Ri - выше.
Всё семейство ВАХов для конкретной лампы состоит из математически неправильных кривых, и чем кривая ближе к оси Ia, тем меньше Ri в её точках при одном и том же значении Ia.

Спасибо: 0 
ПрофильЦитата Ответить



Пост N: 184
Info: -
Зарегистрирован: 27.04.16
Откуда: Россия, Смоленск
Рейтинг: 0
ссылка на сообщение  Отправлено: 15.06.16 15:26. Заголовок: Пермяк пишет: Попыт..


Пермяк пишет:
 цитата:
Попытаюсь ещё раз.

Да, чисто теоретически, согласен.

Можно ли где, найти первоначальный источник данного решения?

И, по какому закону "сопротивление, проявлямое лампой в динамическом режиме", это движение рабочей точки по нагрузочной линии осуществляется?
http://akadem-nauki.ru/dinamicheskij-rezhim-raboty-bpt-dinamicheskaya-xarakteristika/ только это такое же решение построенное по одним вычислениям, а сам закон?

- Не обещайте горы, всё равно подарите камушки. Спасибо: 0 
ПрофильЦитата Ответить
moderator




Пост N: 4112
Зарегистрирован: 15.01.10
Откуда: Россия
Рейтинг: 18
ссылка на сообщение  Отправлено: 15.06.16 16:42. Заголовок: Представим себе, что..


Представим себе, что лампа полностью заперта. Ток через неё равен нулю. Тогда равен нулю и ток через Ra. Падения напряжения на Ra нет, и всё напряжение питания Ea приложено к аноду. Имеем точку с координатами (Ua=Ea; Ia=0), она лежит на оси напряжений.

Теперь представим себе, что лампа полностью открыта. Через неё идёт ток теоретически максимальный, ограниченный только сопротивлением Ra:
Ia.мах=Еа/Ra. Имеем вторую точку, с координатами (Ua=0; Ia=Ia.maх).

Соединяем полученные две точки прямой линией (нагрузка активная), получаем траекторию перемещения рабочей точки.



Таким образом, раб. точка перемещается по линейному ЗАКОНУ... э-э ... в данной системе координат

Спасибо: 0 
ПрофильЦитата Ответить



Пост N: 188
Info: -
Зарегистрирован: 27.04.16
Откуда: Россия, Смоленск
Рейтинг: 0
ссылка на сообщение  Отправлено: 15.06.16 17:18. Заголовок: Пермяк пишет: Предс..



 цитата:
"Теперь представим себе, что лампа полностью открыта...."

, по теоретическому вычислению, да. но практически этого никогда нет.
линия Ra должна иметь совокупность и пропорцию Амплитуда-время.

вот на графике видно, что красная линия нагрузки это совокупность и пропорция Амплитуда-время.
А применение величины Ri по отрезку А-С, вносит искажение по времени.


- Не обещайте горы, всё равно подарите камушки. Спасибо: 0 
ПрофильЦитата Ответить
moderator




Пост N: 4113
Зарегистрирован: 15.01.10
Откуда: Россия
Рейтинг: 18
ссылка на сообщение  Отправлено: 15.06.16 17:34. Заголовок: B_Aleko пишет: 1. Э..


B_Aleko пишет:
 цитата:
Это справедливо при нулевом потенциале сетки и катода. И в этом случае, Ri = (Ua=Ea)/(Ia=0) =290k. Если сразу задать смещение, то этих величин не будет

Ну что Вы такое пишете!
Мы строим нагр. прямую для переменного тока, а катод соединён с землёй накоротко по перем. току через конденсатор Ск.
Падающее на Rк ПОСТОЯННОЕ напряжение в построении не участвует, просто надо после всех вычислений добавить к Еа падающее на Rк постоянное напряжение.

Спасибо: 0 
ПрофильЦитата Ответить



Пост N: 190
Info: -
Зарегистрирован: 27.04.16
Откуда: Россия, Смоленск
Рейтинг: 0
ссылка на сообщение  Отправлено: 15.06.16 17:39. Заголовок: только переменного т..


только переменного тока не будет без постоянных составляющих, которые и задают режим переменному току.


вот потому и интересуюсь, откуда взялось вычислении величины Ri?

- Не обещайте горы, всё равно подарите камушки. Спасибо: 0 
ПрофильЦитата Ответить
moderator




Пост N: 4114
Зарегистрирован: 15.01.10
Откуда: Россия
Рейтинг: 18
ссылка на сообщение  Отправлено: 15.06.16 18:22. Заголовок: B_Aleko пишет: олько..


B_Aleko пишет:
 цитата:
олько переменного тока не будет без постоянных составляющих, которые и задают режим переменному току.

Не надо писАть банальных лозунгов, на моём последнем графике отражены ВСЕ постоянные составляющие.
Только Вы их не видите.
И что показывает Ваш последний график? Что такого нового и неожиданного?

Просьба: читайте и обдумывайте свои посты перед их отправкой!

Спасибо: 0 
ПрофильЦитата Ответить



Пост N: 195
Info: -
Зарегистрирован: 27.04.16
Откуда: Россия, Смоленск
Рейтинг: 0
ссылка на сообщение  Отправлено: 15.06.16 18:46. Заголовок: скажет кто, где взят..


скажет кто, где взять источник: откуда взялось вычислении величины Ri?

- Не обещайте горы, всё равно подарите камушки. Спасибо: 0 
ПрофильЦитата Ответить



Пост N: 267
Зарегистрирован: 06.04.13
Рейтинг: 0
ссылка на сообщение  Отправлено: 15.06.16 19:16. Заголовок: :sm57: Вся тема - т..


Вся тема - троллинг.

Учиться никогда не поздно, и никому не вредно. Спасибо: 0 
ПрофильЦитата Ответить



Пост N: 196
Info: -
Зарегистрирован: 27.04.16
Откуда: Россия, Смоленск
Рейтинг: 0
ссылка на сообщение  Отправлено: 15.06.16 19:32. Заголовок: Ученик пишет: Вся т..


Ученик пишет:
 цитата:
Вся тема - троллинг

и где вы видели провокацию с моей стороны? если что то новое исследуете и изучаете, разве вы не рассматриваете возможные аспекты решений? или как все, под копирку.
может и в жизни так же, что господин дал, то и правильно? сколько всего нового кругом. разве не пытливые люди это придумали? так бы и сидели в каменном веке.
не нравится - не читайте.

- Не обещайте горы, всё равно подарите камушки. Спасибо: 0 
ПрофильЦитата Ответить



Пост N: 268
Зарегистрирован: 06.04.13
Рейтинг: 0
ссылка на сообщение  Отправлено: 16.06.16 02:54. Заголовок: B_Aleko пишет: и гд..


B_Aleko пишет:
 цитата:
и где вы видели провокацию с моей стороны?

Троллинг - более широкое определение, чем примитивная провокация...
 цитата:
если что-то новое исследуете и изучаете

Это "новое" (для Вас, возможно) давно исследовано и изучено, надо лишь учебники почитать.
 цитата:
разве не пытливые люди это придумали

Пытливые люди интересуются, спрашивают, читают литературу, вникают в ответы... а не выдумывают свои определения, ещё и упорствуют в них. Чего стоит очередное открытие в электротехнике и математике:

B_Aleko пишет:
 цитата:
Это справедливо при нулевом потенциале сетки и катода. И в этом случае, Ri = (Ua=Ea)/(Ia=0) =290k.

Оказывается, 290В можно разделить на "0"мА, и при этом получить 290 килоом! Во как!
Это и есть обещанная в стартпосте "масса интересных вычислений по этой теме и другим"?
По Вашей писанине может быть два вывода: или Вы неспособны понять самых простых вещей, или Вы - тролль.

Учиться никогда не поздно, и никому не вредно. Спасибо: 0 
ПрофильЦитата Ответить
постоянный участник




Пост N: 1154
Зарегистрирован: 28.09.09
Откуда: Украина, Черновцы
Рейтинг: 9
ссылка на сообщение  Отправлено: 16.06.16 05:17. Заголовок: B_Aleko пишет: А пр..


B_Aleko пишет:
 цитата:
А применение величины Ri по отрезку А-С, вносит искажение по времени.

О каких "искажениях по времени" может идти речь в безинерционной системе?! Только не упоминайте нанопикосекунды (нано - слово затасканное и подорванное Сколковым).

Я перестал под влиянием Пермяка писать о своем отношении к "выводам" топикстартера, но Ученик абсолютно прав.
Вместо того, чтобы сделать за это время несколько макетов усилителей, послушать музыку и определиться в своих предпочтениях... впрочем, когда-то было сказано, что русский человек может сделать все. Если не будет задумываться об устройстве мироздания.

Спасибо: 0 
ПрофильЦитата Ответить
moderator




Пост N: 4115
Зарегистрирован: 15.01.10
Откуда: Россия
Рейтинг: 18
ссылка на сообщение  Отправлено: 16.06.16 10:19. Заголовок: B_Aleko пишет: скаже..


Ученик пишет:
 цитата:
Оказывается 290В можно разделить на "0"мА, и при этом получить 290 килоом! Во как!

Ну, обмишурился товарищ, с "новаторами" это частенько случается, а Вы - сразу за руку его хватать... :)

B_Aleko пишет:
 цитата:
скажет кто, где взять источник: откуда взялось вычислении величины Ri?

А.М. Бонч-Бруевич. Применение электронных ламп в экспериментальной физике. М.,1956., стр. 68-71.

Здесь попроще:
Р.Лэнди и др. Справочник радиоинженера, 1961, стр. 39.


Спасибо: 0 
ПрофильЦитата Ответить



Пост N: 199
Info: -
Зарегистрирован: 27.04.16
Откуда: Россия, Смоленск
Рейтинг: 0
ссылка на сообщение  Отправлено: 16.06.16 18:18. Заголовок: ALSS пишет: Добавлю..


ALSS, Вы думаете, я просто занят одними вычислениями? ошибаетесь. жаль только то, что у меня нет осциллографа и спектроанализатора, или спектролаба. по звучанию отличия ощутимы. при сборке "стандартным методом, наугад (Ra от величины 1...10Ri)" - ну звучит, ну относительно громко. нет обоснованности значения (Ra от величины 1...10Ri). отсюда и нет прямой зависимости от нагрузки.

- Не обещайте горы, всё равно подарите камушки. Спасибо: 0 
ПрофильЦитата Ответить



Пост N: 200
Info: -
Зарегистрирован: 27.04.16
Откуда: Россия, Смоленск
Рейтинг: 0
ссылка на сообщение  Отправлено: 16.06.16 18:28. Заголовок: Пермяк пишет: А.М. ..


Пермяк пишет:
 цитата:
А.М. Бонч-Бруевич. Применение электронных ламп в экспериментальной физике. М.,1956., стр. 68-71.

а я и не знал, что у меня есть способность к математическим вычислениям.

Давайте так, слегка подведу к истине.
(мю), величина не постоянная, как утверждено - это статический, то есть максимальный параметр лампы при Ia=const., это сказано: Рейх. Теория и применения электронных приборов. стр. 81.; его значение Uвх=м. то есть (дельта), не "d", (дельта)=мю. Чем больше входной сигнал по амплитуде, тем больше мю, но ограничен он только (дельта)Ug. стр. 83.

Далее, в книге: А.М. Бонч-Бруевич. Применение электронных ламп в экспериментальной физике. М.,1956., стр. 70., обратите внимание на формулу 1.87, из нее вытекают следующие 1.88 и 1.89. Обратите внимание на значение Uc=const. Вот, как сказал Иисус: "Имеющий глаза, да не видит...", или по нашему: "Смотрим в книгу, видим фигу". Эта величина "Uc=const" работает только в самих ее пределах!, она устанавливает правила для значений Ua в соотношении с Ia, не БОЛЕЕ! Кто ввел измерение значения Ri, в теорию Бонч-Бруевича???


- Не обещайте горы, всё равно подарите камушки. Спасибо: 0 
ПрофильЦитата Ответить
moderator




Пост N: 4116
Зарегистрирован: 15.01.10
Откуда: Россия
Рейтинг: 18
ссылка на сообщение  Отправлено: 16.06.16 19:19. Заголовок: B_Aleko пишет: (мю),..


B_Aleko пишет:
 цитата:
(мю), величина не постоянная

При первом же взгляде на семейство ВАХ любой лампы любому должно быть ясно, что все три параметра - S, мю, и Ri - величины НЕ постоянные. Но для этого, как минимум, надо знать, как они определяются. Вам очень долго, на 9 страницах, пытались объяснить, что такое эти "дельты", но Вы никак не хотели это понимать.

Есть ТЕОРИЯ, и есть практические МЕТОДЫ РАСЧЁТА, основанные на теории.
Методы расчёта во многих случаях бывают сознательно упрощены для уменьшения трудоёмкости расчётных работ. Погрешность в этих случаях (с учётом разброса параметров ламп) как правило, бывает вполне достаточной для производства аппаратуры, почему и применяются инженерами, и не только ими.

ЗЫ. Это хорошо, что Вы нашли книгу Г.Дж.Рейха, вот посмотрите в ней, как надо рисовать вид сигнала во времени на графике:
http://shot.qip.ru/00Qqtx-6gbw3dFgI/
Только рисунок этот - единственно для пояснения. Потому что, глядя на нагрузочную прямую триода, и без того видно, что правая полуволна тока всегда короче левой.

Спасибо: 0 
ПрофильЦитата Ответить



Пост N: 201
Info: -
Зарегистрирован: 27.04.16
Откуда: Россия, Смоленск
Рейтинг: 0
ссылка на сообщение  Отправлено: 16.06.16 19:32. Заголовок: а что же мне здесь в..


а что же мне здесь все доказывали, что "мю" - постоянна и не меняется? да же график есть:


Вот если бы потом дописали: при выбранных значениях режима лампы, тогда бы все стало на места.

- Не обещайте горы, всё равно подарите камушки. Спасибо: 0 
ПрофильЦитата Ответить
moderator




Пост N: 4117
Зарегистрирован: 15.01.10
Откуда: Россия
Рейтинг: 18
ссылка на сообщение  Отправлено: 16.06.16 20:01. Заголовок: B_Aleko пишет: а что..


B_Aleko пишет:
 цитата:
а что же мне здесь все доказывали, что "мю" - постоянна и не меняется?

Никто Вам такого не писАл.
Я, например, постоянно в постах писал: для рабочей точки, для выбранной точки, для рассматриваемой точки.
А на стр. 9, пост 4068, выложил даже график из ж. РАДИО для 6н3п с изменениями параметров лампы. Только Вы не захотели его осознать.
http://f5.s.qip.ru/q7I7mMvP.png

Спасибо: 0 
ПрофильЦитата Ответить



Пост N: 203
Info: -
Зарегистрирован: 27.04.16
Откуда: Россия, Смоленск
Рейтинг: 0
ссылка на сообщение  Отправлено: 16.06.16 20:07. Заголовок: Да, видел. но все та..


Да, видел. но все таки утверждения что она постоянна - были.

- Не обещайте горы, всё равно подарите камушки. Спасибо: 0 
ПрофильЦитата Ответить
moderator




Пост N: 4118
Зарегистрирован: 15.01.10
Откуда: Россия
Рейтинг: 18
ссылка на сообщение  Отправлено: 16.06.16 20:28. Заголовок: B_Aleko пишет: но вс..


B_Aleko пишет:
 цитата:
но все таки утверждения что она постоянна - были.

Покажите, где. .

Спасибо: 0 
ПрофильЦитата Ответить



Пост N: 205
Info: -
Зарегистрирован: 27.04.16
Откуда: Россия, Смоленск
Рейтинг: 0
ссылка на сообщение  Отправлено: 16.06.16 20:36. Заголовок: Почему в радиотракта..


Почему в радиотрактах приемников, при нагрузке только на индуктивность с малым сопротивлением, применяются пентоды? ведь там дело в этой же величине Ri.

,...а если триод, то там добавочное сопротивление?


- Не обещайте горы, всё равно подарите камушки. Спасибо: 0 
ПрофильЦитата Ответить
moderator




Пост N: 4119
Зарегистрирован: 15.01.10
Откуда: Россия
Рейтинг: 18
ссылка на сообщение  Отправлено: 16.06.16 20:45. Заголовок: B_Aleko , если хотит..


Индуктивное сопротивление катушки на радиочастотах достаточно велико.
На добавочном активном сопротивлении (резисторе) потеряем часть напряжения, развиваемое лампой.

Спасибо: 0 
ПрофильЦитата Ответить



Пост N: 207
Info: -
Зарегистрирован: 27.04.16
Откуда: Россия, Смоленск
Рейтинг: 0
ссылка на сообщение  Отправлено: 16.06.16 22:18. Заголовок: Пермяк пишет: Индук..


Пермяк пишет:
 цитата:
Индуктивное сопротивление катушки на радиочастотах достаточно велико.

а как же режим по постоянному току? катушка имеет очень низкое сопротивление и от этого мал ток. но что бы этот ток был, нужно приложить достаточно низкое входное напряжение. иначе лампа "умрет" при большем напряжении раскачки. верно? может не так разъясняюсь?


- Не обещайте горы, всё равно подарите камушки. Спасибо: 0 
ПрофильЦитата Ответить
moderator




Пост N: 4121
Зарегистрирован: 15.01.10
Откуда: Россия
Рейтинг: 18
ссылка на сообщение  Отправлено: 16.06.16 22:36. Заголовок: B_Aleko пишет: а как..


B_Aleko пишет:
 цитата:
а как же режим по постоянному току?

Так же, как и в трансформаторном каскаде-драйвере. Есть и каскады ЗЧ с дросселем в аноде.


Спасибо: 0 
ПрофильЦитата Ответить



Пост N: 209
Info: -
Зарегистрирован: 27.04.16
Откуда: Россия, Смоленск
Рейтинг: 0
ссылка на сообщение  Отправлено: 17.06.16 04:03. Заголовок: Так все же, какой пр..


Пермяк, у вас был пост №4110, там график с линиями А и С. можете сказать величины Ri, dIa, dUa? кое что проясняется по книге Дж. Рейха. заранее благодарен.

Так все же, какой правильный расчет Ri на графике?

- Не обещайте горы, всё равно подарите камушки. Спасибо: 0 
ПрофильЦитата Ответить
moderator




Пост N: 4122
Зарегистрирован: 15.01.10
Откуда: Россия
Рейтинг: 18
ссылка на сообщение  Отправлено: 17.06.16 08:51. Заголовок: На графике пост 4110..


Величины dUa и dIa - это дифференциальные (бесконечно малые) приращения напряжения и тока анода в точке А. Поскольку бесконечно малые отрезки на графике отразить невозможно, пользуются отрезками реальной длины, более-менее удобной для построения. Таков, например, отрезок АС на ВАХе, и отрезки его проекций на оси координат, которые в отличие от дифферециальных величин, обозначают греческой буквой "дельта": ∆Ua и ∆Ia.

На графике пост 4110 http://hiend.borda.ru/?1-13-0-00000496-000-180-0#216 выполнено одно из возможных построений для определения Ri в конкретно выбранной точке А. Под графиком - в тексте - расчёт.
Прочтите внимательно!
Повторюсь: это не теоретические выкладки, а практический расчёт.


Спасибо: 0 
ПрофильЦитата Ответить



Пост N: 212
Info: -
Зарегистрирован: 27.04.16
Откуда: Россия, Смоленск
Рейтинг: 0
ссылка на сообщение  Отправлено: 18.06.16 02:59. Заголовок: Начну с "хвоста&..


Начну с "хвоста", так как рыбу чистят именно с хвоста. а где начало, "голова", история об этом умалчивает.
Только прошу, без "разных" высказываний. Как смогу, буду объяснять, так как не владею математическим языком.
Книга немецкого инженера, проживавшего в америке: Werner W. Diefenbach, 1954г.













А вот график:

,

при выборе параметра Ri, исходят из точек А и С, но так как по формуле Ri=∆Ua/∆Ia, получается, что это точки ВС. И вычисляется этим образом по книге R(штрих), которое равно анодному сопротивлению.
а Ri, не что иное, как Ri=Ua/Io (по книге).
Исходя из данных, могу предположить, что Ri, не что иное как: изменение участка цепи внутреннего сопротивления лампы, при изменении действующих приращений переменных составляющих тока и напряжения.
И все дело в том, что книг-оригиналов Бонч-Бруевича нет. Только начиная с 1960гг. и то, если присмотреться и прочитать между строк, становится очевидным, что кто то хорошо отредактировал данные его труды. А что касается трудов: Рейх. Теория и применения электронных приборов. 1948-1949гг. стр. 84. дифференциальное уравнение., то вполне может быть закономерностью с учетом времени, после выступления Черчиля на конгрессе.

- Не обещайте горы, всё равно подарите камушки. Спасибо: 0 
ПрофильЦитата Ответить



Пост N: 213
Info: -
Зарегистрирован: 27.04.16
Откуда: Россия, Смоленск
Рейтинг: 0
ссылка на сообщение  Отправлено: 18.06.16 03:09. Заголовок: Далее. Дело в том, ..


Далее.
Дело в том, что немцы всегда отличались своей скурпулезностью и педантизмом.
Принимаем во внимание тот факт, что значение Ri будет равно отрезку линии сопротивления Ra, смотрим второй график,где видно, что дельты Ri равны Ra в точках DE. :




- Не обещайте горы, всё равно подарите камушки. Спасибо: 0 
ПрофильЦитата Ответить



Пост N: 214
Info: -
Зарегистрирован: 27.04.16
Откуда: Россия, Смоленск
Рейтинг: 0
ссылка на сообщение  Отправлено: 18.06.16 03:21. Заголовок: Ближе к сути: Добави..


Ближе к сути:
Добавим к графику еще линии смещения и дополнительные точки статических параметров:



Становится почти очевидным, что статические параметры, точки G,J,K,L., для расчетов нам не нужны. Так как если брать их во внимание, то значения Ri, Ra будут значительно отличаться от расчетных.
далее...

- Не обещайте горы, всё равно подарите камушки. Спасибо: 0 
ПрофильЦитата Ответить



Пост N: 215
Info: -
Зарегистрирован: 27.04.16
Откуда: Россия, Смоленск
Рейтинг: 0
ссылка на сообщение  Отправлено: 18.06.16 03:43. Заголовок: книга Горшков, Справ..


книга Горшков, Справочник Радиолюбителя, 1937г.
отрывки из книги:


где сказано, что величина Ri - внутреннее уравнение лампы, но никак не внутреннее сопротивление по переменному току. И такое определение, что это "внутреннее сопротивление по переменному току" ни где не встречается в технической литературе до 1937 года.

- Не обещайте горы, всё равно подарите камушки. Спасибо: 0 
ПрофильЦитата Ответить





Пост N: 1098
Зарегистрирован: 20.08.13
Откуда: Россия, Москва
Рейтинг: 2
ссылка на сообщение  Отправлено: 18.06.16 03:55. Заголовок: B_Aleko пишет: где ..


B_Aleko пишет:
 цитата:
где сказано, что величина Ri - внутреннее уравнение лампы

Там это не сказано, читайте внимательно. Внутреннее уравнение лампы:
μ = S*Ri

Спасибо: 0 
ПрофильЦитата Ответить



Пост N: 216
Info: -
Зарегистрирован: 27.04.16
Откуда: Россия, Смоленск
Рейтинг: 0
ссылка на сообщение  Отправлено: 18.06.16 03:58. Заголовок: Отсюда: Ri, вычисляе..


Отсюда: Ri, вычисляется из отрезка АС или АВ (dU/dI), или полное приращение из отрезка ВС (∆Ua/∆Ia). последнее и является Ra.


Stan Marsh пишет:
 цитата:
Там это не сказано, читайте внимательно. Внутреннее уравнение лампы:

Там не сказано, что оно является внутренним сопротивлению лампы по переменному току.

И ещё вопрос.
Из курса геометрии: Отрезок — это часть прямой линии, которая ограничена двумя точками (концами отрезка). У отрезка есть и начало, и конец. Длина отрезка — это расстояние между его концами.
Как может половина отрезка являться полным сопротивлением лампы?

Существует такая величина как Ro - величина внутреннего сопротивления постоянной составляющей режима покоя при заданных величин смещения. но куда ее применить? никто не знает.
Этому значению есть прямое применение в решении параметров ламп. но об этом уже завтра.

- Не обещайте горы, всё равно подарите камушки. Спасибо: 0 
ПрофильЦитата Ответить
moderator




Пост N: 4125
Зарегистрирован: 15.01.10
Откуда: Россия
Рейтинг: 18
ссылка на сообщение  Отправлено: 18.06.16 08:45. Заголовок: B_Aleko пишет: Сущес..


B_Aleko пишет:
 цитата:
Существует такая величина как Ro - величина внутреннего сопротивления постоянной составляющей режима покоя при заданных величин смещения.
но куда ее применить?
никто не знает.

Так уж и никто?
Более правильный перевод на русский технический язык: Rо - сопротивление лампы постоянному току в точке покоя, выбранной при заданных Ucм и Ua.
И Вам уже неоднократно писали, что эта физ. величина на практике не нужна (мой пост 4110).
 цитата:
Там не сказано, что оно является внутренним сопротивлению лампы по переменному току

. Термины "по постоянному току" и "по переменному току" - не точные, при рассмотрении работы лампы они примененяются для упрощения. Лично Вас они просто сбивают с толку.
Более точно надо говорить: "в статическом режиме", "в динамическом режиме". Т. е. при отсутствии усиливаемого сигнала, и при его наличии.
 цитата:
Как может половина отрезка являться полным сопротивлением лампы?

Ну, во-первых, Эта ПОЛОВИНА отрезка не ЯВЛЯЕТСЯ, а всего лишь отображает сопротивление.
А во-вторых, и я Вам писал (пост 4110), да и на страничке в Вашем посте 215 сказано, что Ri - это отношение ИЗМЕНЕНИЯ анодного напряжения к ИЗМЕНЕНИЮ анодного тока.

Тут, друг мой, надо смотреть с точки зрения математики.
В системе координат, где гориз. ось - напряжение Uа, а вертикальная ось - ток Iа, сопротивление R выражается не ДЛИНОЙ отрезка, а отношением проекций отрезка на оси координат. Поэтому, если это отрезок ПРЯМОЙ линии, то - независимо от его длины - отношение проекций всегда одинаковое:
R=ΔUa/ΔIa
Очень жаль, что Вы этот важнейший момент до сих пор не поняли. Трудно, конечно, "не для средних умов..." (с)

И гораздо всё будет интереснее, если там будет не прямая, а кривая, да ещё математически неправильная, каковыми являются ВАХи ламп.

Спасибо: 0 
ПрофильЦитата Ответить



Пост N: 222
Info: -
Зарегистрирован: 27.04.16
Откуда: Россия, Смоленск
Рейтинг: 0
ссылка на сообщение  Отправлено: 18.06.16 14:55. Заголовок: Пермяк пишет: И гор..


Пермяк пишет:
 цитата:
И гораздо всё будет интереснее, если там будет не прямая, а кривая, да ещё математически неправильная, каковыми являются ВАХи ламп.

Да, все гораздо будет интереснее дальше. И как вы верно заметили про значение R=ΔUa/ΔIa (не Ri) на графике. Точка В - это рабочая точка. Что это значение R=ΔUa/ΔIa есть на самом деле?
 цитата:
И Вам уже неоднократно писали, что эта физ. величина на практике не нужна (мой пост 4110).

не спешите с выводами. все по порядку.

- Не обещайте горы, всё равно подарите камушки. Спасибо: 0 
ПрофильЦитата Ответить



Пост N: 223
Info: -
Зарегистрирован: 27.04.16
Откуда: Россия, Смоленск
Рейтинг: 0
ссылка на сообщение  Отправлено: 18.06.16 16:12. Заголовок: Продолжаю. Есть одно..


Продолжаю. Есть одно, большое замечание, по тому, как строят графики и указывают на них входные и выходные величины. к чему это?

Все абсолютно уверены, что лампа делает фазовый сдвиг входного сигнала. Я думаю, что этого не происходит, судя по данным ниже графиков. Может кто поможет потом точно разобраться.
Вот здесь можете забросать меня камнями, удалить все мои сообщения и прочее............................. Конечно можете, но наберитесь терпения. Так это тоже не мной придумано.
Это скан из книги:





Как могу, объясню. Дело в том, по представлениям что рисуем, входной сигнал - мы размещаем ниже оси абсцисс, ток - справа оси ординат, а выходной сигнал - на "биссектрисе" линий смещения.

Может кто то объяснит, как возможно в таком случае, что линия сеточного смещения является "зеркалом" для вектора амплитуды. И вдобавок увеличивать значение вектора напряжения выше точки Ua.max?
А где же анодная нагрузочная линия? Вот она то и выполняет распределение (стекания, не отражения) тока-напряжения на своем отрезке, обусловленном точками А и В. (ну не математик я. как могу...)
график 4:



если я окажусь не прав о фазовом сдвиге, заранее простите. Но в том, что на этом графике точно указаны векторы направления сигналов - факт.

Обратите внимание на значение нагрузочной линии: RL=25000 Ом.

- Не обещайте горы, всё равно подарите камушки. Спасибо: 0 
ПрофильЦитата Ответить



Пост N: 224
Info: -
Зарегистрирован: 27.04.16
Откуда: Россия, Смоленск
Рейтинг: 0
ссылка на сообщение  Отправлено: 18.06.16 17:12. Заголовок: смотрим на американс..


смотрим на американский график. его значения:

Ua.min = 92.5
Ua.max = 217.5
Ia.max = 0.0103
Ia.min = 0.0053
ΔUa = Ua.max - Ua.min = 217.5 - 92.5 = 125
ΔIa = Ia.max - Ia.min = 0.0103 - 0.0053 = 0.005
dUa = ΔUa / 2 = 62.5
dIa = ΔIa / 2 = 0.0025

Вычисляем анодное нагрузочное:

Ra = ΔUa / ΔIa = 125 / 0.005 = 25000

Найдем рабочую точку, так как на графике ее точных значений нет.

Uo = Ua.max - dUa = 217.5 - 62.5 = 155 = Uo = Ua.min + dUa = 93.5 + 62.5 = 155
Io = Ia.max - dIa = 0.103 - 0.0025 = 0.0078 = Io = Ia.min + dIa = 0.0053 + 0.0025 = 0.0078

И вот последнее недостающее звено:

Ro = Uo / Io = 155 / 0.0078 = 19872

Определяем напряжение питания:

Ea = (Ro + Ra) x Io = (19872 + 25000) x 0.0078 = 350.0016

Все величины в формулах - по постоянному току!

Смотрим на график.....
Только одно но. Эти значения абсолютно справедливы для паспортных (заводских) ВАХ. Проверено не только на 6Н3П , но и на лапах 6Н2П, 6Н8С, 6Н9С, 6Н23П, 12ax7
На графиках снятых "вручную", достоверность может оказаться плохой.
Как то так, СПАСИБО.

- Не обещайте горы, всё равно подарите камушки. Спасибо: 0 
ПрофильЦитата Ответить



Пост N: 225
Info: -
Зарегистрирован: 27.04.16
Откуда: Россия, Смоленск
Рейтинг: 0
ссылка на сообщение  Отправлено: 18.06.16 17:27. Заголовок: И последнее дополнен..


И последнее дополнение:
Если я хочу усиление =20, выбираю 6Н23П
РТ со значениями:

Uo = 70
Io = 0.0095
Imax = 0.014
Imin = 0.0065
Umax = 90
Umin = 50

Вычисляю:
Ro = 7368
Ra = 5333
Ea = 120

график для сравнения


- Не обещайте горы, всё равно подарите камушки. Спасибо: 0 
ПрофильЦитата Ответить





Пост N: 1100
Зарегистрирован: 20.08.13
Откуда: Россия, Москва
Рейтинг: 2
ссылка на сообщение  Отправлено: 18.06.16 17:34. Заголовок: B_Aleko пишет: Если..


B_Aleko пишет:
 цитата:
Если я хочу усиление =40, выбираю 6Н23П

Не получится. μ не позволит.

Спасибо: 0 
ПрофильЦитата Ответить
постоянный участник




Пост N: 1156
Зарегистрирован: 28.09.09
Откуда: Украина, Черновцы
Рейтинг: 9
ссылка на сообщение  Отправлено: 18.06.16 17:44. Заголовок: B_Aleko пишет: Учен..


B_Aleko пишет:
 цитата:
Все абсолютно уверены, что лампа делает фазовый сдвиг входного сигнала.

Чиво?! В интересующей нас области звуковых частот лампа абсолютно безинерционный прибор. Чего не скажешь о каскаде, в котором она используется.
Не обобщайте свое незнание на весь мир и тем более не выдавайте свое "прозрение" за мировое откровение.

Спасибо: 0 
ПрофильЦитата Ответить



Пост N: 230
Info: -
Зарегистрирован: 27.04.16
Откуда: Россия, Смоленск
Рейтинг: 0
ссылка на сообщение  Отправлено: 18.06.16 17:55. Заголовок: Уважаемые форумчане!..


Уважаемые форумчане!
Это НЕ мои собственные мысли! Это было придумано почти сотню лет назад.
Если вы считаете все описанное "бредом, ахинеей, вымыслом", прошу, без оскорбления.
Попробуйте сами разобраться в имеющемся и решить для себя, надо это вам или нет!
Мне НАДО! потому, что это правильно.
СПАСИБО.

- Не обещайте горы, всё равно подарите камушки. Спасибо: 0 
ПрофильЦитата Ответить



Пост N: 273
Зарегистрирован: 06.04.13
Рейтинг: 0
ссылка на сообщение  Отправлено: 18.06.16 17:57. Заголовок: B_Aleko пишет: Это ..


B_Aleko пишет:
 цитата:
Это НЕ мои собственные мысли! Это было придумано почти сотню лет назад.

Учебники, которые Вы предъявляете - да. Но выводы, которые Вас "озаряют", неверные.
И самое плохое - Ваше нежелание прислушаться. В конце-концов, если Вы так уверены в своей правоте, получается, что оппонирующие Вам ничего не понимают? Все разом?
Обычно бывает по-другому: если у кого-то "все вокруг дураки", это значит, что дурак всего один.

Учиться никогда не поздно, и никому не вредно. Спасибо: 0 
ПрофильЦитата Ответить



Пост N: 231
Info: -
Зарегистрирован: 27.04.16
Откуда: Россия, Смоленск
Рейтинг: 0
ссылка на сообщение  Отправлено: 18.06.16 18:00. Заголовок: Ученик пишет: Все р..


Ученик пишет:
 цитата:
Все разом?

Вы за всех не говорите.

P.S. Пусть оценят, просчитают. А там примут решение.
до вечера, с детьми еду отдыхать!

- Не обещайте горы, всё равно подарите камушки. Спасибо: 0 
ПрофильЦитата Ответить



Пост N: 274
Зарегистрирован: 06.04.13
Рейтинг: 0
ссылка на сообщение  Отправлено: 18.06.16 19:19. Заголовок: B_Aleko пишет: Вы з..


B_Aleko пишет:
 цитата:
Вы за всех не говорите

Сторонников Ваших выкладок не видать.
 цитата:
Пусть оценят, просчитают.

Так Вы уже всё "просчитали", и даже сделали вывод:
 цитата:
... это правильно.


Зачем Вам другие мнения?

Учиться никогда не поздно, и никому не вредно. Спасибо: 0 
ПрофильЦитата Ответить





Пост N: 1973
Info: Начинающий любитель.
Зарегистрирован: 04.05.11
Откуда: Россия, Уфа
Рейтинг: 3
ссылка на сообщение  Отправлено: 21.03.19 20:32. Заголовок: Почитал, вроде много..


Почитал тему, вроде многое понятно, но все же есть вопросы
В частности, допустим я выбрал рабочую точку для лампы в трансформаторном каскаде.
А что в таком случае будет при изменении анодной нагрузки? Сама точка же не меняется, если изменилась только нагрузка?

Спасибо: 0 
ПрофильЦитата Ответить
постоянный участник


Пост N: 19211
Зарегистрирован: 04.04.09
Рейтинг: 52
ссылка на сообщение  Отправлено: 21.03.19 21:59. Заголовок: подрежет вашу точку ..


подрежет вашу точку с одной стороны.Точку мы выстраиваем на конкретной нагрузке по симметричному ограничению и далее по наичистому спектру.

Страница Бокарёва
http://tubeaudio.ucoz.ru/index/stranica_aleksandra_bokarjova/0-51
Спасибо: 0 
ПрофильЦитата Ответить





Пост N: 1975
Info: Начинающий любитель.
Зарегистрирован: 04.05.11
Откуда: Россия, Уфа
Рейтинг: 3
ссылка на сообщение  Отправлено: 21.03.19 23:20. Заголовок: Сорри, я имел в виду..


Сорри, я имел в виду увеличение нагрузки относительно оптимальной.

Спасибо: 0 
ПрофильЦитата Ответить
постоянный участник


Пост N: 19220
Зарегистрирован: 04.04.09
Рейтинг: 52
ссылка на сообщение  Отправлено: 21.03.19 23:21. Заголовок: Ограничение сдвинетс..


Ограничение сдвинется в другую сторону

Страница Бокарёва
http://tubeaudio.ucoz.ru/index/stranica_aleksandra_bokarjova/0-51
Спасибо: 0 
ПрофильЦитата Ответить





Пост N: 1976
Info: Начинающий любитель.
Зарегистрирован: 04.05.11
Откуда: Россия, Уфа
Рейтинг: 3
ссылка на сообщение  Отправлено: 22.03.19 01:54. Заголовок: Бокарёв Александр С..


Бокарёв Александр
Странно, почему тогда расчет по ВАХ показывает минимальные искажения, просто мощность меньше?

Спасибо: 0 
ПрофильЦитата Ответить
постоянный участник




Пост N: 3634
Зарегистрирован: 19.02.06
Откуда: Россия
Рейтинг: 9
ссылка на сообщение  Отправлено: 22.03.19 02:05. Заголовок: Jaster пишет: А что..


Jaster пишет:
 цитата:
Странно, почему тогда расчет по ВАХ показывает минимальные искажения, просто мощность меньше?

Почему же минимальные? Он показывает в основном искажения по второй гармонике и не факт, что они минимальные в выбранной Вами точке.
Вам к истокам нужно.Такое впечатление, что Вы немного не понимаете как это всё работает.
Сюда: http://ulfdiysound.ucoz.ru/load/nachinajushhemu_audio_samodelshhiku/3

Спасибо: 0 
ПрофильЦитата Ответить





Пост N: 206
Зарегистрирован: 11.05.15
Откуда: Израиль, Ариель
Рейтинг: 1
ссылка на сообщение  Отправлено: 22.03.19 03:32. Заголовок: Jaster пишет: А что..


Jaster пишет:
 цитата:
А что в таком случае будет при изменении анодной нагрузки? Сама точка же не меняется, если изменилась только нагрузка?

В трансформаторном каскаде, конечно, от смены нагрузки на вторичке положение самой точки покоя на ВАХ не меняется. Ток и напряжение покоя на аноде, как были, так и останутся.

В резистивном каскаде смена анодного резистора меняет и положение раб. точки.

Спасибо: 0 
ПрофильЦитата Ответить





Пост N: 1981
Info: Начинающий любитель.
Зарегистрирован: 04.05.11
Откуда: Россия, Уфа
Рейтинг: 3
ссылка на сообщение  Отправлено: 22.03.19 04:26. Заголовок: Вот. теперь сомнения..


Вот. теперь сомнения рассеялись. :) Спасибо всем.

Спасибо: 0 
ПрофильЦитата Ответить
Ответов - 246 , стр: 1 2 3 4 5 6 7 8 9 10 11 12 13 All [только новые]
Ответ:
1 2 3 4 5 6 7 8 9
большой шрифт малый шрифт надстрочный подстрочный заголовок большой заголовок видео с youtube.com картинка из интернета картинка с компьютера ссылка файл с компьютера русская клавиатура транслитератор  цитата  кавычки моноширинный шрифт моноширинный шрифт горизонтальная линия отступ точка LI бегущая строка оффтопик свернутый текст

показывать это сообщение только модераторам
не делать ссылки активными
Имя, пароль:      зарегистрироваться    
Тему читают:
- участник сейчас на форуме
- участник вне форума
Все даты в формате GMT  8 час. Хитов сегодня: 611
Права: смайлы да, картинки да, шрифты да, голосования нет
аватары да, автозамена ссылок вкл, премодерация откл, правка нет